Published by AMT PUBLISHiNG Mathernafics Trust University of Canberra ACT 2601 AUSTRALIA Copyright C2001 AMT Publishing Telephone: +61 2 6201 5137 AMTT Limited ACN 083 950 341 National Library of Australia Card Number and ISSN Australian Mathematics Trust Enrichment Series 1SSN 1 326-0170 101 Problems in Algebra iSBN 1 876420 12 X THE AUSTRALIAN MATHEMATICS TRUST ENRiCHMENT SERIES MiTT E EJ • • Chairman Editor GRAHAM H POLLARD, Canberra AUSI'RALiA PETER J TAYLOR, Canberra AUsTRALiA WARREN J ATKiNs, Canberra AUSTRALIA Eo J BARBEAU, Toronto CANADA GEORGE BERZSENYI, Terra Haute USA RON DUNKLEY, Waterloo CANADA WALTER E MIENTKA, lincoln USA N1KOLAY K0NsrANT1N0v, Moscow RussiA ANDY Liii, Edmonton CANADA JORDAN B TABOV, Sofia BULGARiA JOHN WEBB, Cape Town SouFH AFRiCA The books in this series are selected for their motivating, interesting and stimulating sets of quality problems, with a lucid expository style in their solutions. Typically, the problems have occurred in either national or international contests at the secondary school level. They are intended to be sufficiently detailed at an elementary level for the mathematically inclined or interested to understand but, at the same time, be interesting and sometimes challenging to the undergraduate and the more advanced mathematician. lt is believed that these mathematics competition problems are a positive influence on the learning and enrichment of mathematics. PREFACE This book contains one hundred highly rated problems used in the training and testing of the USA international Mathematical Olympiad (IMO) team. It is not a collection of one hundred very difficult, impenetrable questions. Instead, the book gradually builds students' algebraic skills and techniques. This work aims to broaden students' view of mathematics and better prepare them for possible participation in various mathematical competitions. It provides in-depth enrichment in important areas of algebra by reorganizing and enhancing students' problem-solving tac- tics and strategies. The book further stimulates students' interest for future study of mathematics. INTRODUCTION In the United States of America, the selection process Leading to par- ticipation in the International Mathematical Olympiad (IMO) consists of a series of national contests called the American Mathematics Contest 10 (AMC 10), the American Mathematics Contest 12 (AMC 12), the American Invitational Mathematics Examination(AIME), and the United States of America Mathematical Olympiad (USAMO). Participation in the AIME and the USAMO is by invitation only, based on performance in the preceding exams of the sequence. The Mathematical Olympiad Summer Program (MOSP) is a four-week. intense training of 24-30 very promising students who have risen to the top of the American Mathematics Competitions. The six students representing the United States of America in the IMO are selected on the basis of their USAMO scores and further IMO-type testing that takes place during MOSP. Throughout MOSP, full days of classes and extensive problem sets give students thorough preparation in several important areas of mathematics. These topics include combinatorial arguments and identities, generating functions, graph theory. recursive relations, telescoping sums and products, probability, number theory, polynomials, theory of equations. complex numbers in geometry, algorithmic proofs, combinatorial and advanced geometry, functional equations and classical inequalities. Olympiad-style exams consist of several challenging essay problems. Cor- rect solutions often require deep analysis and careful argument. Olympiad questions can seem impenetrable to the novice, yet most can he solved with elementary high school mathematics techniques, cleverly applied. Here is some advice for students who attempt the problems that follow. • Take your time! Very few contestants can solve all the given problems. • Try to make connections between problems. A very important theme of this work is; all important techniques and ideas featured in the book appear more than once! • Olympiad problems don't "crack" immediately. Be patient. Try different approaches. Experiment with simple cases. In some cases, working backward from the desired result is helpful. • Even if you can solve a problem, do read the solutions. They may contain some ideas that did not occur in your solutions, and they VIII may discuss strategic and tactical approaches that can be used else- where. The formal solutions are also models of elegant presentation that you should emulate, but they often obscure the torturous process of investigation, false starts, inspiration and attention to detail that led to them. When you read the solutions, try to reconstruct the thinking that went into them. Ask yourself. "What were the key ideas?" 4How can I apply these ideas further?" • Go back to the original problem later, and see if you can solve it in a different way. Many of the problems have multiple solutions, but not all are outlined here. • All terms in boldface are defined in the Glossary. Use the glossary and the reading list to further your mathematical education. • Meaningful problem solving takes practice. Don't get discouraged if you have trouble at first. For additional practice, use the books on the reading list. ACKNOWLEDGEM ENTS Thanks to Tiankai Liu who helped in proof reading and preparing solu- Many problems are either inspired by or fixed from mathematical contests in different countries and from the following journals: High-School Mathematics, China Revista Maternaticã Romania Kvant, Russia cite all the original sources of the problems in the solution part. We express our deepest appreciation to the original proposers of the problems. We did our best ABBREVIATIONS AND NOTATIONS Abbreviations AHSME AIME AMC1O AMC12 ARML IMO USAMO MOSP Putnam St. Petersburg American High School Mathematics Examination American Invitational Mathematics Examination American Mathematics Contest 10 American Mathematics Contest 12, which replaces AIISME American Regional Mathematics League International J\Iathematical Olympiad United States of America Mathematical Olympiad Mathematical Olympiad Summer Program The William Lowell Putnam Mathematical Competition St. Petersburg (Leningrad) Mathematical Olympiad Notations for Numerical Sets and Fields Z N N0 Q Q+ Q° R IR+ R° R'2 C the set of integers the set of integers modulo n the set of positive integers the set of nonnegative integers the set of rational numbers the set of positive rational numbers the set of nonnegative rational numbers the set of n-tuples of rational numbers the set of real numbers the set of positive real numbers the set of nonnegative real numbers the set of n-tuples of real numbers the set of complex numbers C 1 • PREFACE vii • INTRODUCTION ix • ACKNOWLEDGEMENTS xi • ABBREViATiONS AND NOTATiONS • 1. INTRODUCTORY PROBLEMS • 2. ADVANCED PROBLEMS xiii 1 13 • 3. SOLUTIONS TO INTRODUCTORY PROBLEMS 27 • 4. SOLUTIONS TO ADVANCED PROBLEMS 65 • GLOSSARY 131 • FURTHER READING 137 iNTRODUCTORY PROBLEMS INTRODUCTORY PROBLEMS 1 problem 1 Let a. b. and c be real and positive parameters. Solve the equation ax = CX-r Problem 2 Find the genera! terni of the sequence = for bi + ax + by = 3. - cx. x1 = — alln E N. Problem 3 x,, be a sequence of integers such that Let x1. x2 n: (i) (ii) x1 + x2 + + = 19: =99. (iii) Determine the minimum aiid niaxiniuni possible values of Problem 4 The function f. defined by 1(x) where a. b. c. and h = c.r+ d d are nonzero real nunibers. has the j)rOperties f(19) = for ax + 19. all values of x. eXCCl)t Find range of f, f(97) = -- 97. and f(f(x)) = and 1. Introductory Problems 2 Problem 5 Prove that (a—b)2 < 8a for all a a+b — (a—b)2 2 8b — b> 0. Problem 6 Several (at least two) nonzero numbers are written on a board. One may erase any two numbers. say a and b, and then write the numbers a + and b — instead. Prove that the set of numbers on the board, after any number of the preceding operations, cannot coincide with the initial set. Problem 7 The polynomial may be written in the form ao + a1y + a2y2 where y = x + 1 and Find a2. + a15y16 + a17y17, are constants. Problem 8 Let a, b, and c be distinct nonzero real numbers such that a+-=b+-c 1 1 b Prove that = 1 a 1. Problem 9 Find polynomials 1(x), g(x), and h(x), if they exist, such that for all x, ( —1 3x+2 —2x+2 ifx<—1 if—i ifx>0. i. Introductory Problems Problem 10 Find all real numbers x for which 8x+27x 7 12x + 18x Problem 11 Find the least positive integer m such that (2n'\ I I for all positive integers n. Problem 12 Let a, b, c, d, and e be positive integers such that Find the maximum possible value of max{a, b, c, d, e}. Problem 13 Evaluate 3 4 2001 1!+2!+3! 1999!+2000!+2001! Problem 14 Letx= 1, a ER. Find all possible values of x. Problem 15 Find all real numbers x for which + lix + 12T = 13X + 14x. 1. Introductory Problems 4 Problem 16 Let f N x N N be a function such that f(1, 1) = 2, f(rn + 1. n) = f(rn. n) + rn and f(rn, n + 1) = f(n2, n) — n for all rn, n E N. Find all pairs (pq) such that f(p,q) = 20th. Problem 17 Let f be a function defined on [0, 1] such that f(0) = for all a b 1(1) and (f(a) — f(b)I < a— bI. 1 in the interval [0, 1]. Prove that 1 1(a) — f(b)( < Problem 18 Find all pairs of integers (x, y) such that x3 + y3 = (x + y)2. Problem 19 2 Let f(x) = for real numbers x. 4x + 2 Evaluate 1 / \ 1 2 /2000 •\ Problem 20 Prove that for ii 6 the equation 1 1 x1 x2 1 has integer solutions. Problem 21 Find all pairs of integers (a, b) is divisible by x2 — x — 1. such that the polynomial ax'7 + bx16 + I 1. IntroductorY Problems Problem 22 Given a positive integer n, let p(n) be the product of the non-zero digits of n. (If n has only one digit, then p(n) is equal to that digit.) Let +p(999). What is the largest prime factor of S? Problem 23 Let be a sequence of nonzero real numbers such that xn = — n = 3,4 Establish necessary and sufficient conditions on x1 and x2 for for to be an integer for infinitely many values of ri. Problem 24 Solve the equation — 3x = Problem 25 For any sequence of real numbers A = {ai,a2,as,. the sequence {a2 — a1, a3 — a2, a4 — a3.. of the sequence Find a1. . . }, define to be }. Suppose that all of the terms are 1, and that a1g = a92 = 0. Problem 26 Find all real numbers x satisfying the equation 2x+3x 4X+6X9X =1. Problem 27 Prove that 16 <17. Problem 28 Determine the number of ordered pairs of integers (rn n) for which mn m3 + n3 + 99mn = 1. Introductory Problems 6 Problem 29 Let a, b, and c be positive real numbers such that a + b + c 4 and ab 4-bc+ca 4. Prove that at least two of the inequalities a—bl 2, lb—cl 2, Ic—al are true. Problem 30 Evaluate 1 Problem 31 Let 0 < a < 1. Solve = ax for positive numbers x. Problem 32 What is the coefficient of x2 when is expanded? Problem 33 n be distinct positive integers. Find the maximum value of lxm — where x is a real number in the interval (0,1). Let Problem 34 Prove that the polynomial are distinct integers, cannot be written as the product of two non-constant polynomials with integer coefficients, i.e., it is where ai. a2, .. irreducible. , Problem 35 Find all ordered pairs of real numbers (x. y) for which: (1+x)(1+x2)(1+x4) and (1+y)(1+y2)(1+y4) = 1+x7. Problem 36 Solve the equation 2(2x — 1)x2 + (2z2 — 2)x = 2 for real numbers x. Problem 37 Let a be an irrational number and let n be an integer greater than 1. Prove that is an irrational number. Problem 38 Solve the system of equations = x2(x4+x5—x2) (x1 — x2 -I-- x3)2 = x3(x5+x1—x3) = (x4 — x5 + x1)2 = x5(x2+x3—x5) (x5 — xi + = x1(x3+x4—x1) (x2 — x3 + x4)2 (x3 — x4 + x5)2 for real numbers Xl, x2, x4, x5. Problem 39 Let x, y, and z be complex numbers such that x y +z= 2, + z2 = + and STAATS.UL.NIV) 11 = 3 4. Xy+z—1 + yz+x—1 + zx+y—1 — 1. Introductory Problems 8 Problem 40 Mr. Fat is going to pick three non-zero real numbers and Mr. Taf is going to arrange the three numbers as the coefficients of a quadratic equation _x2 +_x+_=O. Fat wins the game if and distinct rational solutions. Who has a winning strategy? Mr. only if the resulting equation has two Problem 41 Given that the real numbers a, b, c, d, and e satisfy simultaneously the relations a+b+c+d+e=8anda2-j-b2+c2+d2+c2 =16, determine the maximum and the minimum value of a. Problem 42 Find the real zeros of the polynomial Pa(S) = (x2 + 1)(x — 1)2 — ax2, where a is a given real number. Problem 43 Prove that 1 3 2n—1 2 4 2n 1 for all positive integers n. Problem 44 Let P(x) = a nonzero polynomial with integer coefficients such that P(r) = P(s) = 0 for some integers r and s. with 0 < r < s. Prove that ak < —s for some k. Problem 45 Let rn be a given real number. Find all complex numbers x such that (S \ (X\ / +( 2 =m +772. Problem 46 The sequence given by xo = a, .r1 = b, and 11 1 is periodic. Prove that ab = 1. Problem 47 Let a, b, c, and d be real numbers such that (a2 +b2 — i)(c2 +d2 —1)> (ac+bd— 1)2. Prove that a2 + b2> 1 and c2 + d2> 1. Problem 48 Find all complex numbers z such that (3z + 1)(4z + 1)(6z + 1)(12z + 1) = 2. Problem 49 be the zeros different from 1 of the polynomial Let P(x) = — 1, n 2. Prove that 1 i—x1 + 1 i—x2 +...+ n—i 1 2 Problem 50 Let a and b be given real numbers. Solve the system of equations V1_x2+y2 for real numbers x and y. = a — b ADVANCED PROBLEMS Z ADVANCED PROBLEMS Problem 51 Evaluate (2000'\ (2000\ (2000\ (2000 Problem 52 Let x, y, z be positive real numbers such that x4 + y4 -i- z4 = Determine with proof the minimum value of x3 1—x 8+ l—y8 z3 ± 1—z8 Problem 53 Find all real solutions to the equation 2X + 3X + 6X = x2. Problem 54 Let {an}ni be a sequence such that = 2 and 1 = +— for all n E N. Find an explicit formula for Problem 55 Let x, y, and z be positive real numbers. Prove that 2; + + z z+ y) 1. 1. 2. Advanced Problems 14 Problem 56 Find, with proof, all nonzero polynomials f(z) such that f(z2) + f(z)f(z +1) =0. Problem 57 Let f N N be a function such that f(n + 1) > f(n) and f(f(n)) = 3n for all n. Evaluate 1(2001). Problem 58 Let F be the set of all polynomials f(x) with integers coefficients sueh that f(x) = 1 has at least one integer root. For each integer k > 1. find the least integer greater than 1 for which there exists f E P such that the equation f(x) = rnk has exactly k distinct integer roots. Problem 59 Let x1 = 2 and = — ÷ 1, i. Prove that for ii 1 1— Problem 60 Suppose that f 22 1 1 .r1 x2 1 1 22 is a decreasing function such that for all : x,y E f(x + y) + f(f(x) + 1(y)) = 1(1(1 + f(y)) + f(y + 1(x))). Prove that f(f(x)) = x. 15 Problem 61 Find all functions f : Q Q such that f(x + y) + f(x for — y) = 2f(x) + 2f(y) all x,y EQ. Problem 62 Let <a < 1. Prove that the equation x3(x + 1) = (x + a)(2x + a) has four distinct real solutions and find these solutions in explicit form. Problem 63 Let a, b, and c be positive real numbers such that abc = Prove that 1 1 I <1. + ± 1. 1 a+b+1 c+a+1 b+c-I-1 Problem 64 Find all functions f, defined on the set of ordered pairs of positive integers, satisfying the following properties: f(x, x) = x, f(x, y) = f(y, x). (x + y)f(x. y) yf(x. x + y). Problem 65 Consider n complex numbers zk. such that IzkI 2 1, Ic = Prove that there exist c1. a2.... E f—i. 1} such that, for any vi e1z1 + + Problem 66 Find a triple of rational numbers (a. + <2. c) such that n. fl, 2. Advanced 16 Problem 67 Find the minimum of + log12 (X2-_ log11 + — where x1,x2,. .. — are real numbers in the interval 1). Problem 68 Determine x2 + + z2 + w2 if 22_12+ 22_32+22_52 =1, +4252 =1, 42_12 + 42_32 62_12 +6232+6252+6272 =L 82_12 + 82_32 -L 82_52 + 82_72 =1 Problem 69 Find all functions f R : R such that f(xf(x) + f(y)) = (f(x))2 + for all x,y E Problem 70 The numbers 1000, iOOl, ,2999 have been written on a board. Each time, one is allowed to erase two numbers, say, a and b, and replace them by the number min(a. b). After 1999 such operations, one obtains exactly one number c on the board. Prove that c < 1. Problem 71 Let a1,a2,. . be real numbers, not all zero. Prove that the equation . has at most one nonzero real root. 2. Advanced Problems Problem '72 real numbers defined by a1 = t and Let {an) be the sequence of = for n 1. For bow many distinct values oft — do we have a1998 = 0? Problem '73 R (a) Do there exist functions f : f(g(x)) = x2 and and g R R such that g(f(x)) = for all x E R? and 9: (b) Do there exist functions f: R f(g(x)) = x2 and R g(f(x)) such that = for all x E R? Problem 74 LetO<a1 a2 < O<b1 <b2 Suppose that there exists 1 b2 for i > k n such berealnumberssuchthat for 1 that i k and k. Prove that a1a2 b1b2 Problem 75 Given eight non-zero real numbers a1, a2,. a8. prove that at least one of the following six numbers: a1a3 + a2a4, a1a5 + a2a6, a1a7 + a2a8, a3a5 + a4a6, a3a7 + a4a8. + a6a8 is non-negative. Problem '76 Let a, b and Prove that c be positive real numbers such that abc ab bc ca = 1. a5+b5+ab+b5+cS+bc+c5+a5+caL 2. Advanced Problems 18 Problem 77 Find all functions f: R —* R such that the equality f(f(x) + y) = f(x2 y) — + 4f(x)y holds for all pairs of real numbers (x. y). Problem 78 Solve the system of equations: 3x — y x2 — + =3 y2 x+3y 11 =0 Problem 79 Mr. Fat and Mr. Taf play a game with a polynomial of degree at least 4: +... +_x + 1. They fill in real numbers to empty spaces in turn. If the resulting polynomial has no real root, Mr. Fat wins; otherwise, Mr. Taf wins. If Mr. Fat goes first, who has a winning strategy? Problem 80 Find all positive integers k for which the following statement is true: F(x) is a polynomial with integer coefficients satisfying the condition for then F(0)= F(1)= c=0.1 k+1, =F(k+ 1). Problem 81 The Fibonacci sequence is given by (nEN). Prove that 2n+2 ' for all n 2. 2n—2 bf2n if 19 ced ProblemS Problem 82 R for which there exists a strictly monotonic Find all functions u : function f : R —p R such that f(x + y) = f(x)u(y) + f(y) for all x, y c R. Problem 83 Zn be Let complex numbers such that IzlI+Iz2H+IzflI=1. Prove that there exists a subset S of {z1, Z2,.. . . such that Problem 84 A polynomial P(x) of degree n 5 with integer coefficients and n distinct integer roots is given. Find all integer roots of P(P(x)) given that 0 is a root of P(x). Problem 85 Two real sequences x1, x2, and Yi . Y2. . are defined in the following way: and Yn+1 for all n 1. Prove that 2 < yn = 1 + yi + <3 for all a > 1. Problem 86 For a polynomial P(x), define the difference of P(x) on the interval [a. b] ([a,b), (a,b). (a.b]) as P(b) P(a). Prove that it is possible to dissect the interval [0. 1[ into a finite number of intervals and color them red and blue alternately such that. for every polynomial P(x). the total difference of P(.r) on red intervals is equal to that of P(x) on blue intervals. What about cubic polynomials? - 2. Advanced Problems 20 Problem 87 Given a cubic equation x3 + _x2 + _x + — = 0, Mr. Fat and Mr. Taf are playing the following game. In one move, Mr. Fat chooses a real number and Mr. Taf puts it in one of the empty spaces. After three moves the game is over. Mr. Fat wins the game if the final equation has three distinct integer roots. Who has a winning strategy? Problem 88 Let n > 2 be an integer and let f R2 any regular n-gon A1A2 . R be a function such that for . . f(A1) + f(A2) + . + f is the zero function. Problem 89 Let p be a prime number and let f(x) be a polynomial of degree d with integer coefficients such that: (i) f(0) = 0, f(l) = 1; (ii) for every positive integer n, the remainder upon division of by p is either 0 or 1. Prove that d p — 1. Problem 90 Let n be a given positive integer. with a0 = Consider the sequence a0,a1,• ak = 4 TI fork=l,2..n, Prove that 1 _! TI < 1. and 2. 21 Advanced problems Problem 91 Let ai,a2,." be nonnegative real numbers. not all zero. — (a) Prove that positive real root R. (b) Let A = a2 and Prove that AA — — = 0 has precisely one ja3. B= <RB. Problem 92 Prove that there exists a polynomial P(x, y) with real coefficients such that P(x, y) 0 for all real numbers .r and y. which cannot be written as the sum of squares of polynomials with real coefficients. Problem 93 For each positive integer Ti. show that there exists a positive integer k such that /c = f(x)(x + 1)2n + + 1) for some polynomials f.g with integer coefficients, and find the smallest such Ic as a function of n. Problem 94 Let x be a positive real number. (a) Prove that (x (ii— 1)! 1 1).. (x + n) — I . (b) Prove that (n—i)! 1 2. Advanced 22 Problem 95 Let ii 3 be an integer, and let Xcs={1.2 n3} be a set of 3n2 elements. Prove that one can find nine distinct numbers such that the system c7, = 0 a2x+h2y+c2z = 0 a3x+b3y+c3z 0 = (1 = 1. 2, 3) in X has a solution (XO, yo, zO) in nonzero integers. Problem 96 he positive real numbers. Let n 3 be an integer and let Xi. i2. Suppose that = 1 Prove that Problem 97 Let x1,x2 be distinct real numbers. Define the polynomials P(x) = (x — x1)(.r (1 and Q(x)=P(x)( \.i—X1 + •.. — 1 X—X2 -+. Let y1,y2.. . . 'Yn—l be the roots of Q. Show tJ < — + that 2. Advanced Problems Problem 98 Show that for any positive integer fl. the polynomial f(x) = (x2 + +1 cannot be written as the product of two non-constant polynomials with integer coefficients Problem 99 be functions such that R Let 11,12.13 R a1f1 + a2]'2 + a3f3 monotonic for all a1,a2,a3 Prove that there exist c1, c2. C3 is R. R. not all zero. such that cifi(x) +c2f2(x) for all x +c3f3(x) = 0 Ift. Problem 100 be the sums of be variables, and let Let x1.x2, nonempty subsets of elementary symmetric polynomial in be the Let pk(xl the yj (the sum of every product of A- distinct yjs). For which k and n is every coefficient of Pk (as a polynomial in x1,. .. . . , even? For example, if n = 2. Pi = P2 = P3 = + s2 and then yi' Y2. y3 are Xl. = + Y2 + Y1Y2 + 2x2, + 112Y3 + Y3Y1 = Y1Y2Y3 = X1X2 + + + Problem ioi. Prove that there exist 10 distinct real numbers al,a2.. the equation (x—ai)(x_a2)...(xaio) has exactly 5 different real roots. = (X+al)(X+a2) . aio such that (x+aio) SOLUTiONS TO iNTRODUCTORY PROBLEMS 3. SOLUTIONS TO PROBLEMS problem 1 [Romania 1974] Let a, b, and c be real and positive parameters. Solve the equation \/c+ax= v'b—ax -f-s/c— Solution 1 It is easy to see that x = 0 is a solution. Since the right hand side is a decreasing function of x and the left hand side is an increasing function of x, there is at most one solution. Thus x = 0 is the only solution to the equation. Problem 2 Find the general term of the sequence defined by x0 = _2 — 3, = 4 and — for all n C N. Solution 2 We shall prove by induction that = n + 3. The claim is evident for n=O,1. Fork 1, if Xk_1 =k+2andxk=k+3, then —kxk =(k+2)2 —k(k+3)=k+4, as desired. This completes the induction. 3. Solutions to Introductory Problems 28 Problem 3 [AHSME 1999] Let x1. x2, , x7, be a sequence of integers such that . . n; (i) (ii) 19; X1 (iii) Determine the minimum and maximum possible values of Solution 3 Let a, b, and c denote the number of —is, is, and 2s in the sequence, respectively. We need not consider the zeros. Then a, b, c are nonnegative integers satisfying —a+b+2c= l9anda+b+4c=99. It follows that a = 40—c and b 59—3c, where 0 c 19 (since b 0), so 19+6c. (a = 40,b = 59), the lower bound (19) is achieved. When c = 19 (a = 21, b = 2), the upper bound (133) is achieved. When c = 0 Problem 4 [AIME 1997] The function f, defined by f(x) ax + b cx+d' where a, b, c, and d are nonzero real numbers, has the properties 1(19) = 19, f(97) = 97, and for all values of x, except — Find the range of f. Solution 4, Alternative 1 For all x, f(f(x)) = .r, i.e., (ax + b'\ a( \cx+d) (ax+b\ cx + dj —x f(f(x)) = to Introductory Problems 3 i.e. 29 (a2 + bc)x + b(a + d) c(a+d)x+bc+d2 i.e. — X. c(a + d)x2 + (d2 — a2)x — b(a + d) = 0, implies that c(a + d) = 0. Since c 0, we must have a = —d. The conditions f(19) = 19 and f(97) = 97 lead to the equations which and Hence (972 It follows that a = 58c, 192)c — 972c=2•97a+b. = 2(97 — 19)a. which in turn leads to b = — 1843 f(x)= 58x.x—58 —1843c. Therefore 1521 which never has the value 58. Thus the range of f is R — {58}. Solution 4, Alternative 2 The statement implies that f is its own inverse. The inverse may be found by solving the equation ay + b cy + d fory. Thisyields dx—b —cx + a The nonzero numbers a. b, c, and d must therefore be proportional to d, —c, and a, respectively; it follows that a = —d, and the rest is the same as in the first solution. Problem 5 Prove that forallab> (a—b)2 < 8a a+b 2 0. Alternative 1 that ) - (a—b)2 — 8b 3. Solutions to Introductory Problems 30 i.e. 4a 4b — i.e. (a—b)2 < < (a—b)2 from which the result follows. Solution 5, Alternative 2 Note that (a-t-b\2 2 -ab + (a-b)2 2(a + b) + Thus the desired inequality is equivalent to 4a a+b+ which is evident as a b> 0 (which implies a 4b. b). Problem 6 [St. Petersburg 1989] Several (at least two) nonzero numbers are written on a board. One may erase any two numbers, say a and b, and then write the numbers a + and b — instead. Prove that the set of numbers on the board. after any number of the preceding operations, cannot coincide with the initial set. Solution 6 Let S be the sum of the squares of the numbers on the board. Note that S increases in the first operation and does not decrease in any successive operation, as a)2 = (a+ with equality only if a = b = This completes the proof. 0. to Introductory Problems 31 problem 7 [AIME 1986] The may be written in the form ao + aiy + a2y2 + . + ai6y'6 + a17y17, - are constants. Find a2. = x + 1 and SolutiOn 7, Alternative 1 where y Let f(x) denote the given expression. Then xf(x)=x—x2+x3— .—x18 and (1 + x)f(x) = 1 Hence f(x)=f(y—1)= — 1— (y— 1)18 — 1— (y— 1)18 l+(y—l) y Therefore a2 is equal to the coefficient of y3 in the expansion of (y 1 — — 1)18, i.e., a2 = 816. = Solution 7, Alternative 2 Let 1(x) denote the given expression. Then f(x) =f(y—1) l—(y— l)-f(y— Thus /2\ /3\. (17\ Here we used the I and the fact that 7i '\ /n+1" (18" 3. Solutions to Introductory Problems 32 Problem 8 Let a. b, and c be distinct nonzero real numbers such that -1 = b+ -1 = c+ -.1 c b a Prove that = 1, Solution 8 From the given conditions it follows that b—c c—a a—b a—b=—.b—c———,andc—a=—. bc ca ab Multiplying the above equations gives (abc)2 = result follows. 1, from which the desired Problem 9 [Putnam 1999] Find polynomials f(s), g(x), and h(s). if they exist. such that for all x — g(')I + h(s) = ( ifr<—1 —1 3x + 2 —2x+2 if —1 < .r ifx>0. < 0 Solution 9, Alternative 1 Since x = —1 and x = 0 are the two critical values of the absolute functions, one can suppose that F(s) = = I (c—a—b)x+d—a ifs <—1 if—1<x<0 1(a+b+c)x+a+d ifx>0. which implies that a 3/2, b = —5/2. c = —1. and d = 1/2. Hence f(s) = (3s + 3)/2, g(x) = 5x/2, and h(s) = —x + Solution 9, Alternative 2 Note that if r(x) and 8(x) are any two functions, then max(r,s) r-l-s+lr—sj = 2 Therefore, if F(s) is the given function. we have F(x) = max{—3x — 3. 0} — max{5x, 0} + 3x + 2 = (—3x—3-'-j3x+31)/2—(5x+15x1)/2-4-3x--i-2 = colutions to Introductory Problems 33 Problem 10 Find all real numbers x for which 8X — 7 12x + 18x — 6 Solution 10 = = a and By setting the equation becomes b. a3+b3 _7 a2b+ b2a — i.e. 6 a2—ab+b2 _7 6' ab i.e. Ga2 — l3ab+6b2 = 0. i.e. (2a — 3b)(3a — 2b) = 0. Therefore 2x+i = 3x+i or 2x—1 x = 1. It is easy to check that both .r = = 3x—1, which —1 and x = 1 implies that x = such (2n\ \n) that <ni for all positive integers n. ii Note that + + < ... + and for n (10) Thus in = 252 > and satisfy the given equation. Problem 11 [Romania 1990] Find the least positive integer in —1 = (1+ i)2n = 3. Solutions to Introductory Problems 34 Problem 12 Let a. b, c, d, and e be positive integers such that abcde=a+b+c+d+e. Find the maximum possible value of max{a, b, c. d, e}. Solution 12, Alternative 1 Suppose that a b c d e. We need to find the maximum e. Since value of c<a+b+c+d+c<5c. 5e, i.e. < abcd 5. Hence = (1.1,1,2), (1,1,1,3). (1,1, 1.5), which leads to max{e} = 5. then e < abcdc 1 (1. 1,1,4), (1, 1,2,2), or Solution 12, Alternative 2 As before, suppose that a c b d c. Note that 1 1 1 1 1 1 de d d = 1, then a = b = c = 1 and 4 + c = e, which is impossible. e 5. Thusd—1 land c—i It is easy to see that (1, 1,1,2,5) is a solution. Therefore max{e} = Comment: 5. The second solution can be used to determine the maxiwhen x1, x2, . are positive integers mum value of {x1, x2,. . such . , . . . that x1x2 . = xi+ . + + xm. Problem 13 Evaluate 3 1!+2!+3! + 4 2!+3!+4! ++ 2001 to Introductory Problems 35 13 Note that k+2 = k![1+k+1+(k+1)(k+2)] k+2 1 — k!(lc+2) — (k+2)! — (k+2)—1 — (k-i--2)! k — 1 — (k+1)! — (k+2)! By telescoping sum, the desired value is equal to 2 2001! Problem 14 Find all possible values of x. solution 14, Alternative 1 Since \/a2 + al + 1> al and 2a Va2 +a+ 1+ Va2 -a+ 1' we have lxi < 12a/ai = 2, Squaring both sides of Va2+a+1 Yields — a + 1 = 2a — x2. 3. Solutions to Introductory Problems 36 Squaring both sides of the above equation gives 4(x 2 —1)a =x (x2'—4)ora = 2 2 2 x2(x2—4) 4(21)' Since a2 0, we must have x2(x2 — 4)(x2 — 1) 0, Since xI < 2, x2 —4 < 0 which forces x2 —1 < 0. Therefore, —1 < x < 1. Conversely, for every x C (—1, 1) there exists a real number a such that x Va2 +a+1 — —a+1. Solution 14, Alternative 2 Let A = B= and P = (a.O). Then P is a point on the x-axis and we are looking for all possible values of d=PA-PB. By the Triangle Inequality, IPA — PBJ < = 1. And it is clear that all the values —1 <d < 1 are indeed obtainable. In fact, for such a d, a half hyperbola of all points Q such that QA — QB = d is well defined, (Points A and B are foci of the hyperbola.) Since line AB is parallel to the x-axis, this half hyperbola intersects the x- axis, i.e., P is well defined. Problem 15 Find all real numbers x for which lOx + lix + 12x = 13X + 14x. Solution 15 It is easy to check that x = 2 is a solution. We claim that it is the only one. In fact, dividing by 13X on both sides gives 112\x The left hand side is a decreasing function of x and the right hand side is an increasing function of x. Therefore their graphs can have at most one point of intersection. Solutions to Introductory Problems 37 Comment: More generally. a2+(a+1)2+.+(a+k)2 =(a+k+1)2+(a+k+2)2+"+(a+2k)2 forak(2k+'). kcN. problem 16 [Korean Mathematics Competition 2001] N be a function such that f(1, 1) = 2. Let f: N x N f(m + 1. n) = f(ni, n) + ni and f(772, f(n2. n) — 72 + 1) for all rn, n c N. Find all pairs (p, q) such that f(p. q) = 2001. Solution 16 We have f(pq) = f(p—l,q)+p—1 = = = 2 = f(1,l) = q(q—1) p(p—l) 2 2 2001. Therefore p(p— 1) —q(q— 2 2 1) — 1999, — i.e. (p—q)(p+q— 1) =2. 1999. ROte that 1999 is a prime number and thatp—q < p+q— 1 for p.q We have the following two cases: 1 = 1999. Hence p= 1001 and q= 999. c N 3. Solutions to Introductory Problems 38 Therefore (p. q) = (2000, 1999) or (1001,999). Problem 17 [China 1983] Let f be a function defined on [0, 1] such that f(0) = f(1) = land f(a)—f(b)I < la—bi, bin the interval [0, 1]. for all a Prove that f(a) — f(b)j < Solution 1.7 We consider the following cases. 1. a — bI < 1/2. Then If(a) — f(b)f < a - bI as desired. 2. Ia — bi > 1/2. By symmetry, we may assume that a> b. Then f(a) — f(b)I = If(a) — 1(1) + f(0) — f(b)I < f(a) — f(')I + 1(0) — f(b)I < Ia—lI+I0—bI = 1—a+b—0 = 1—(a-b) as desired. Problem 18 Find all pairs of integers (x, y) such that + = (x + y)2. Solution 18 Since x3+y3 = (x+y)(x2—xy+y2), all pairs of integers (n—ri), n C 7Z. are solutions. Suppose that x + y 0. Then the equation becomes S2 i.e. — Xy + y2 = x + y, 3 solutions to Introductory Problems 39 a quadratic equation in x, we calculate the discriminant A = y2 + 2y + 1 — 4y2 + 4y = —3y2 + 6y + 1. solving for A 0 yields 3 3 Thus the possible values for y are 0, 1, and 2. which lead to the solutions (1,0), (0,1), (1,2), (2.1), and (2,2). Therefore, the integer solutions of the equation are (x. y) = (1,2), (2,1), (2,2), and (n, —n). for all nEZ. (1, 0), (0, 1), Problem 19 [Korean Mathematics Competition 2001] Let 2 f(x)= 4X+2 for real numbers x. Evaluate / 1 \ / 2 /2000 '\ f has a half-turn symmetry about point (1/2, 1/2). Indeed, 2.4x 2 from which it follows that 1(x) f(1 -t- — x) 4X 1. Thus the desired sum is equal to 1000. Problem 20 Prove that for n 6 the equation 1 1 1 x1 has integer solutions. Solution 20 Note that 1 1 1 1 1 3. Solutions to Introductory Problems 40 from which it follows that if (x1, x2, = , (a1 .a2, is an inte. ger solution to 1 1 x1 x2 1 then (x1,x2,. . is an integer solution to 1 1 1 xl x2 Therefore we can construct the solutions inductively if there are solutions for n = 6, 7, and 8. Since xi = 1 is a solution for n = 1, (2,2,2,2) is a solution for n = and (2, 2, 2,4,4.4,4) is a solution for n = 7. 4, It is easy to check that (2, 2, 2,3, 3, 6) and (2, 2. 2, 3,4,4, 12, 12) are solutions for n = 6 and n = 8, respectively. This completes the proof. Problem 21 [AIME 1988] Find all pairs of integers (a, b) such that the polynomial ax17 + bx'6 -i- 1 is divisible by x2 — x — 1. Solution 21, Alternative 1 Let p and q be the roots of x2 — x — I = 0. By Vieta's theorem, p + q = 1 and pq —1. Note that p and q must also be the roots of ax17 + bx'6 + 1 = 0. Thus ap17 + bp'6 = —1 and aq17 + bq'6 —1. Multiplying the first of these equations by q'6, the second one by p16, and using the fact that pq = —1, we find ap+b=—q16andaq+b=—p'6. Thus a= I q'6 = (p8 + q8)(p4 + q4)(p2 + q2)(p ± q). (1) solutions to Introductory Problems 41 since p+q = 1, p2+q2 = (p+q)2—2pq=1+2=3, p4+q4 = (p2+q2)2—2p2q2=9—2=7, (p4+q4)2—2p4q4=49—247, = p8+q8 follows that a 1 3 7 . 47 = 987. Likewise, eliminating a in (1) gives —b = p'7—q'7 p—q = p'6+p'5q+p'4q2++q16 = (p'6 + q16) + pq(p'4 + q14) + p2q2 (p'2 + q'2) = + p7q7(p2 + q2 ) + p8q8 (p16 +q'6) —(p'4 +q'4)+ 2n+4 — = +q2)+ 1. 2n+4 2n+4 P (p2fl+2 = for (p2 k2 = 3 and k4 = 7, and For n 1, let 1. — + + q2Th) + q2) — — n 3. Then k6 18, k8 = 47, k1,, = 123, k,2 = 322, k,4 843, k16 = 2207. Hence —b=2207—843+322—123+47— 18+7—3+1=1597 or (a, b) = (987, —1597). solution 21, Alternative 2 The other factor is of degree 15 and we write (C15x15 — C14X14 + .. + cix — — x— 1) = ax17 + bx'6 + 1. Comparing coefficients: x0: c0=1. c0—c,=0,c,=1 x2: andfor3<k<15, xk: —co—c,+c2=0.c2=2, Ck_2Ck_,+CkO. 3. Solutions to Introductory 42 It follows that for k 15, = Fk+1 (the Fibonacci number). Thus a = —c14 — c15 = —F17 = —1597 = F16 = 987 and b (a, b) = (987, —1597). Comment: Combining the two methods, we obtain some interesting and Since facts about sequences = — = — = — it follows that and satisfy the same recursive relation. easy to check that k2 = F1 + F3 and k4 = F3 + F5. = and Therefore + —I.2m 2n+1 — 2n—2 -rLb2n—4 — Lf 2 It ,i5 L1 Problem 22 [AIME 1994] Given a positive integer n, let p(n) be the product of the non-zero digits of n. (If n has only one digit, then p(n) is equal to that digit.) Let S=p(l)+p(2)+ +p(999). What is the largest prime factor of 8? Solution 22 positive integer less than 1000 to be a three-digit number by prefixing Os to numbers with fewer than three digits. The sum of the products of the digits of all such positive numbers is Consider each (0O.O+OO.1+...+9.99)—O.O.O However, p(n) is the product of non-zero digits of n. products The sum of these can be found by replacing 0 by 1 in the above expression, since ignoring 0's is equivalent to thinking of them as l's in the products. (Note that the final 0 in the above expression becomes a 1 and compensates for the contribution of 000 after it is changed to 111.) Hence S —1 = (46— 1)(462+46+ 1) = 33 5• 7.103, and the largest prime factor is 103. Solutions to Introductory Problems 43 problem 23 [Putnam 1979] of nonzero real numbers such that Zn be a sequence — xn —' Xn_2Xm_1 — for n = 3,4 establish necessary and sufficient conditions on x1 and x2 for an integer for infinitely many values of n. to be Solution 23, Alternative 1 We have — = Let 2 — 1 Xn_2Xn_1 — — = Yn—1 — Yn—2. i.e.. y,-, is an arithmetic is a nonzero integer when n is in an infinite set 8, the Then sequence. If 1. yb's for n C S satisfy —1 Since an arithmetic sequence is unbounded unless the common difference is 0, 7/n — Yn—i = 0 for all n, which in turn implies that x1 = = m, a nonzero integer. Clearly, this condition is also sufficient. Solution 23, Alternative 2 An easy induction shows that — xn_ (n — x1x2 — (n 1)xi — 2)x2 — — x1x2 (x1 — x2)n + (2x2 — x1) for n 3,4 In this form we see that of n if and only if x1 = will be an integer for infinitely many values = in for some nonzero integer m. Problem 24 Solve the equation — 3x = Solution 24, Alternative 1 It is clear that 1. —2 —2. We consider the following cases. x < 2. Setting x = 2cosa, 0 < a < ir, the equation becomes 8cos3 a or — 6cosa = + 1). 3. Solutions to Introductory Problems 44 from which it follows that cos 3a = Then 3a — or 3a + = = 2m7r, rn Since 0 a cos 2n7r, n E Z. the solution in this case is 4ir x=2cos0=2, 2. x> 2. Then x3 — 4x = x(x2 — 4) > 0 4ir andx=2cosT. and x2—x—2=(x—2)(x4-1) >0 or x> It follows that — 3x > x> Hence there are no solutions in this case. Therefore, x = 2, x = 2cos4ir/5, and x = 2cos4ir/7. Solution 24, Alternative 2 For x> 2. there is a real number t > 1 such that x=t 21 The equation becomes (t7 — 1)(t5 — 1) = which has no solutions for t> 1. Hence there are no solutions for x > 2. For —2 x 2, please see the first solution. 0, Solutions to Introductory Problems 45 problem 25 [AIME 1992] sequence of real numbers A = {ai, a2, a3, any For }, define AA to be are 1, and that Suppose that all of the terms of the sequence = Find al a92 — 0. SolutiOn 25 Suppose that the first term of the sequence Then is d. AA={d,d+1,d+2,..} term given by d + (n — with the 1). Hence with term given by the =a1 + (n — 1)d+ — 1)(n —2). a quadratic polynomial in ii with leading coefficient This shows that 1/2, Since a19 = a92 = 0. we must have = — 19)(n —92), soa1 =(1—19)(1 —92)/2=819. Problem 26 [Korean Mathematics Competition 2000] Find all real numbers x satisfying the equation 2X + 3X — 4X + 6X — 9X solution 26 Setting = a and = b, = the equation becomes 1-i-a2 -i-b2 —a —b—ab= 0. Multiplying Square5 gives both sides of the last equation by 2 and completing the (1—a)2+(a—b)2+(b—- 1)2 3. Solutions to Introductory Problem 46 = 3X and x = Therefore 1 = 0 is the only solution. Problem 27 [China 1992] Prove that 16< <17. Solution 27 Note that Therefore which proves the lower bound. On the other hand. Therefore 17. which proves the upper bound. Our proof is complete. Problem 28 [AHSME 1999] Determine the number of ordered pairs of integers (m. m) for which mm O and in3 + n3 + 99mn = Solution 28 Note that (m + n)3 = 333 m3 = (in + n)3 = Hence m + n — 33 + = m3 + n3 + 3mn(m + n). If m + n = 33. then n3 + 3nin(m + n) = in3 + is a factor of in3 + n3 + 99inn — n3 + + 99mm. We have 99mm — 333 n — 33)[(m — m)2 + (m + 33)2 + (n + 33)2] 5olution5 to Introductory Problems Uence 47 there are 35 solutions altogether: (0,33), (1.32), (33,0), and (33, -33). corn ient: More generally, we have a3 + + c3 — 3abc problem 29 [Korean Mathematics Competition 2001] Let a, b, and c be positive real numbers such that a + b c 4 and ab+bc+ca 4. Prove that at least two of the inequalities a—bI2, are true. Solution 29 We have (a+b+c)2 <16. i.e. a2+b2+c2+2(ab+bc+ca)<16, i.e. a2 + b2 -f c2 8, i.e. a2+b2+c2—(ab+bc+ca) 4, i.e. (a—b)24-(b—c)2+(c—a)2 8, and the desired result follows. Problem 30 Evaluate 1 3. Solutions to Introductory 48 Solution 30 denote the desired sum. Then Let s — — (2n)! 1 (2n)!k(n_k)!(n+k)! n 2n 1 ,' 1 (272 — / — (2n)! 2 (2n)! 2(n!) Problem 31 [Romania 1983] Let 0 < a < 1. Solve = aX for positive numbers x. Solution 31 Taking loge yields aX loge x = Consider Then functions from both f f(x)g(x) = f(x)=aX. g(x)=Iogax, h(x)=xa. and are decreasing and h is has unique solution x = a. increasing. h(x) Problem 32 What is the coefficient of x2 when is expanded? It follows that to Introductory Problems 49 3 SolUtbohl 32 Let It = an.o + afl1X+ = is eaSY to see that and 1 Since = = = we (1 + — +...) (1 + 2Thx) 1)x + have + + = a1,2 — — — + 26 + + + 22n) — (22 + =2+ 24(2272_2 — 1) - — 3 — 22Th+2_3.2n+1 +2 — — 3 — + + 1) (2n+1 —1) (2m+1 2) 3 Problem 33 Let in and n be distinct positive integers. Find the maximum value of x is a real number in the interval (0, Solution 33 By symmetry, we can assume that rn > n. Let y = 0 < x < 1, < xTh and 0 < y <1. Thus Since — Xmn) = = APplying the AM-GM inequality yields I — = = — ( ri — _y)m_fl 72 (n. / +(m — — n)(1 n+m—n = — mm 3. Solutions to Introductory 50 Therefore Is —x ( =(m—n)(— — It \ I mm J Equality holds if and only if (rn—n)y 1 or In \ \m Comment: For m = n + 1, we have — 5n+1 < — for real numbers 0 <x <1. Equality holds if and only if x = n/(n + 1). Problem 34 Prove that the polynomial , are distinct integers, cannot be written as the product of two non-constant polynomials with integer coefficients, i.e., it is where a1, a2,• irreducible. Solution 34 For the sake of contradiction, suppose that is not irreducible. Let f(s) = p(x)q(x) such that p(x) and q(x) are two polynomials with integral coefficients having degree less than n. Then g(x) = p(x) + q(x) is a polynomial with integral coefficients having degree less than n. Since p(aj)q(aj) = f(aj) = —1 and both p(aj) and q(aj) are integers, = = 1 to Introductory Problems and Thus 51 p(aj) + q(aj) = 0. g(x) has at least ri roots. But degg <n, so g(x) = 0. p(x) = —q(x) and f(x) = Then —p(x)2, which implies that the leading coefficient of 1(x) must be a negative integer, which is impossible, since the leading coefficient of f(x) is 1. problem 35 Find all ordered pairs of real numbers (x, y) for which: and (1+x)(1+x2)(1+x4) = 1+y7 (1+y)(1+y2)(1+y4) 1-f-x7. 35 We consider the following cases. Solution 1. xy = Then it 0. clear that x = is y = 0 and (x, y) = (0,0) is a solution. 2. xy <0. By the symmetry, we can assume that x > 0 > y. Then (1+x)(1+x2)(1+x4) >1 and 1+y7 <1. Therearenosolutions in this case. 3. x, y > 0 and x y. By the symmetry, we can assume that x > y > 0. Then (1 +x)(1 +x2)(1 +x4) >1 +x7>1 +y7, showing that there 4. x,y <0 and x are no solutions in this case. y. By the symmetry, we can assume that x < 0. Multiplying by 1 — x and 1 — y y < the first and the second equation, respectively, the system now reads 1—x8 = (1+y7)(1—x)=1—x+y7—xy7 1—y8 = (1+x7)(1—y)=1—y+x7—x7y. Subtracting the first equation from the second yields xS_yS=(x_y)-f-(x?_y7)_xy(x6_y6). Since x <y <0, xs — X > 0. > 0, x— y < 0, x7 — y7 <0, —xy (1) <0, and Therefore, the left-hand side of (1) is positive while the right-hand side of (1) is negative. — Thus there are no solutions in this case. 3. Solutions to Introductory Problems 52 5. x= y. Then solving 1— leads to x = 0, 1 x — xy7 1— x + X7 X8 1, —1, which implies that (x, y) = (0.0) or (—1, —1). Therefore, (x, y) = (0,0) and (—1, —1) are the only solutions to the system. Problem 36 Solve the equation — 1)x2 + (2x2 — 2)x = 2x+1 —2 for real numbers x. Solution 36 Rearranging terms by powers of 2 yields 1)—2(x2+x—1) =0. Setting y = —1 (1) and dividing by 2 on the both sides, (1) becomes 2Yx+2xy_(x+y) =0 or (2) Since f(x) = —1 and x always have the same sign, 0. Hence if the terms on the left-hand side of (2) are nonzero, they must have the same sign, which in turn implies that their sum is not equal to 0. Therefore (2) is true if and only if x = x = —1,0, and 0 or y = 0. which leads to solutions 1. Problem 37 Let a be an irrational number and let n be an integer greater than 1. Prove that (a+ Va2_ + (ais an irrational number. to Introductory Problems 3 53 37 SolUtbol1 Let .1. N= (a+ + (a— and let — 1)*, For the sake of contradiction, assume that N b + 1/b. Then by using the identity Then N = rational. = + repeatedly for m m E N. In particular, + - + + 1) we obtain that btm + 1/btm is rational for all 1, 2 =a+ 1+a- Va2 - 1=2a is rational, in contradiction with the hypothesis. Therefore our assumption is wrong and N is irrational. Problem 38 Solve the system of equations (x1 — X2 = x3)2 x2(x4 + X5 — x2) (x2—x3+x4)2 = x3(x5+x1—x3) (x3—x4+x5)2 = = = (x5—x1-f-x2)2 for real numbers x1, x2, x4(x1+x2—x4) x5(x2+x3—x5) xi(x3+x4—x1) x3, x4, S5. 38 Let 5k+5 = Adding the five equations gives 5 5 — is + 2XkXk+2) = + 2XkXk+2). k=1 It follow5 that — = 0. 3. Solutions to Introductory Problems 54 Multiplying both sides by 2 and completing the squares yields — from which x1 = system are x2 = X3 = x4 = 0, Xk+1)2 = Therefore the solutions to the (xi,x2.x3,x4,xs) = (a, a, a. a, a) for a C R. Problem 39 Let x. y, and z be complex numbers such that x+y+z = 2, x2+y2+z2 3, and xyz = 4. Evaluate 1 xy+z—1 + 1 yz+x—l + 1 zx+y—l Solution 39 Let S be the desired value. Note that xy+z—l =xy+1—x--y=(x—l)(y—1). Likewise, yz+x—l (y— 1)(x— 1) zx+y— 1 = (z— 1)(x —1). and Hence 1 S = — - 1 1 (x_1)(y_1)+(y_1)(z_1)+(z_1)(x_1) — —1 x+y+z—3 (x-l)(y-l)(z--l) —1 — xyz—(xy+yz+zx)+x+y+z-- 1 —1 — 5—(xy+yz+zx) But 2(xy + yz + zx) = Therefore S = —2/9. (x + y + z)2 — (x2 + + z2) = 1. 5oiutions to Introductory Problems 55 problem 40 [USSR 1990] Mr Fat is going to pick three non-zero real numbers and Mr. Taf is going 8rrange the three numbers as the coefficients of a quadratic equation x2+ x+ =0. Mr. Fat wins the game if and only if the resulting equation has two distinct rational solutions. Who has a winning strategy? Solution 40 Mr. Fat has the winning strategy. A set of three distinct rational nonzero numbers a, b, and c, such that a + b + c = 0, will do the trick. Let A, B, and C be any arrangement of a, b, and c, and let f(x) = Ax2 + Bx + C. Then f(1) = A + B + C = a+b + c =0. which implies that 1 is a solution. Since the product of the two solutions is C/A, the other solution is C/A. and it is different from 1. Problem 41 [USAMO 1978] Given that the real numbers a, b, c, d, and e satisfy simultaneously the relations a+b+c+d+e=Sanda2+b2+c2+d2+c2= 16, determine the maximum and the minimum value of a. Solution 41, Alternative 1 Since the total of b.c. d. and c is S — a, their average is x = a)/4. (S — Let b=x+b1, c=x+c1, d=x+d1, e=x+c1. Then b1 + d1 + c1 = 0 and (8—a)2 (1) or 0 5a2 — 16a = a(5a — 16). Therefore 0 a 16/5, where a = 0 if and only if b = nd a 16/5 if and only if c—d 6/5 c = d = c = 2 3. Solutions to Introductory Problem5 56 Solution 41, Alternative 2 By the EtMS-AM inequality, (1) follows from and the rest of the solution is the same. Problem 42 Find the real zeros of the polynomial Pa(X) = (x2 + 1)(x where 1)2 — ax2. 2x + 1) — ax2 0. 1'\ x + — I (x —2 + — I — a 0. — a is a given real number. Solution 42 We have (x2 + 1)(x2 — Dividing by x2 yields I I 1\I xj\ xj By setting y = x + 1/x, the last equation becomes y2 —a — = 0. It follows that x+- = x 1± which in turn implies that, if a 0, then the polynomial Pa(X') has the real zeros ± 1+ 2 In addition, if a 8, then Pa(X) also has the real zeros 1- fiPi ± 2 Problem 43 Prove that 1 3 2 4 for all positive integers n. 2n—1 2n 1 Solutions to Introductory Problems 57 1 4 We use induction For n = 1, the result is evident. SuPPOSe the statement is true for some positive integer k, i.e., 2k—i 3 1 1 2k Then 3 2k—1 2k+1 2k+1 1 2k In order for the induction step to pass it suffices to prove that 2k+1 1 I 2k+2 < y3k+4 This reduces to (2k+1\2 3k+1 <3k+4' i.e. (4k2+4k+1)(3k+4) <(4k2+8k+4)(3k+1), i.e. 0< k, which is evident. Our proof is complete. Comment: proved to By using Stirling numbers, the upper bound can be imfor sufficiently large n. Problem 44 [USAMO Proposal, Gerald Heuer] Let P(x) = a nonzero polynomial with integer coefficients such that P(r) = for Some integers P(.s) = r and s, with 0 <r <s. that ak —s for some k. 0 3. Solutions to Introductory 58 Solution 44 Write P(x) = (x — S)XCQ(X) and + ... + Q(x) = b0xm + 0. Since Q has a positive root, by Descartes' rule of signs, either there must exist some k for which bk > 0 bk4l, or bm > 0. If there exists a k for which bk > 0 bk+1, then where bm ak+1 = —sbk + bk+1 —S. If bm > 0, then —S. = —sbm In either case, there is a k such that ak —s, as desired. Problem 45 Let rn be a given real number, Find all complex numbers x such that '.2 '.2 / / IX IX \ \ +I—1 =772 I—1 \x+1J \X—1J +712. 2 Solution 45 Completing the square gives \2 2x2 x—lj = x 2—1 +m2+rn, / x \,x+l i.e. / x 2x2 \2 I = 2x2 x2—1 2 +712 +772. Setting y = 2x2/(x2 — 1), the above equation becomes — y — (rn2 + m) = 0, i.e. (y—m—1)(y+m)=0. Thus 2x2 x2—1 =—772or 2x2 x2—1 =712+1, which leads to solutions -2andx 1. solutions to Introductory Problems 59 problem 46 = The sequence given by xo = a, = b, and +— 2\ — js periodic. prove that ab = 1. solution 46 on both sides of the given recursive relation yields by = +1 or — = 1) — 1. is a geometric — 1 for n C N. Since Yn+i = xn is periodic, then so is which implies that = 0 for all n E N. Therefore = Let sequence. If ab = XçjXl = Yi + 1= 1. Problem 47 Let a, b, c, and d be real numbers such that (a2 + b2 1)(c2 — + d2 —1) > (ac+ bd — 1)2. Prove that a2 + b2> I and c2 + d2 > 1. Solution 47 For the sake of the contradiction, suppose that one of a2 + b2 or c2 + d2 'S less than or equal to 1. Since (ac+bd— 1)2 0, a2 +b2 —land C2 + d2 — 1 must have the same sign. Thus both a2 ± b2 and c2 + d2 are less than 1. Let x=1—a2—b2andy=1—c2---d2. Thefl 0 < x, y 4xy 1. Multiplying by 4 on both sides of the given inequality + 2bd — 2)2 = (2 — 2ac — 2bd)2 = (a2+b2+x-i-c2+d2+y—2ac—2bd)2 = [(a—c)2 > (2ac (x+y)2=x2+2xy+y2, 3. Solutions to Introductory Problems 60 or 0> x2 — 2xy f y2 = (x — y)2, which is irhpossible. Thus our assumption is wrong and both a2 + b2 and c2 + d2 are than 1. Problem 48 Find all complex numbers 2 such that (3z+1)(4z+ 1)(6z+ 1)(12z+ 1) =2. Solution 48 Note that 8(3z = + 1)6(4z + 1)4(6z 4- l)2(12z -1- 1) 768, i.e. +4)(24z+2) = 768. (24z+8)(24z +6)(24z Setting u = 24z + 5 and w = u2 yields 1)(u— 1)(u—3).= 768. (u+3)(u+ i.e. (u2 — 1)(u2 — 9) = i.e. — lOw — 759 = 0, i.e. (w—33)(w+23) Therefore the solutions to the Z 24 =0. given equation are andz= 24 Problem 49 Let x1,x2, P(x) = — Prove that be the zeros different from 1, n 1 of the polynomial 2. 1 1—x1 + 1 1—x2 1 n—i 1- 2 to Introductory Problems 61 Alternative 1 5olutiofl Q(x) = P(1— x) = (1 — — Then Q(x) = (_1)nxnl + and +•• + - are the nonzero roots of the polynomial Q(x). as (1—a—i =0. Thus the desired sum is the sum of .the reciprocals of the roots of polynomial Q(x), that is, 1 1 — X1 + 1 I 1 I — — =—+—+4-— a2 1 1 1 a1 = a2a3 By + ala3 -- + a1a2 the Vieta's Theorem, the ratio between and is equal to the additive inverse of the ratio between the coefficient of x and the constant term in Q(x), i.e., the desired value is equal to (n) as desired. Solution 49, Alternative 2 For any polynomial R(x) of degree n—i, whose zeros are Xi. the following identity holds: 1 X—X1 + 1 +...+ 1 = R'(x) R(x) Xn_ 1' 3. So'utions to Introductory Problems 62 For X—l —2 1 x—1 +...+x+1, R(1) = n and R'(i)=(n— 1)+(n—2)+'+ = n(n— 1) It follows that 1 1—x1 1 1 +...+ 1—x2 + R'(l) n—i R(1) 2 Problem 50 Let a and b be given real numbers. Solve the system of equations — — y — a — s/f— x2 + y2 for real numbers x and y. Solution 50 Let u = x,+ y and v = x — y. Then U+V 2 2 U—V Adding the two equations and subtracting the two equations in the original system yields the new system (a+b)s/1—uv = = Multiplying the above two equations yields uv(1 — uv) hence uv = a2 — b2. = (a2 — b2)(1 — uv). It follows that u= (a+b)i/1—a2+b2 andv= (a—b)V1—a2+b2 which in turn implies that (a+bV'a2_b2 b+a'1a2—b2 -i-b2 whenever 0 < a2 — b2 < 1. SOLUTiONS TO ADVANCED PROBLEMS 4. SOLUTIONS TO ADVANCED PROBLEMS problem 51 f2000\ f2000\ 1 2000\ (2000 Solution 51 Let 2000 1(x) = (1 + x)200° = Let w = (—1 Then w3 = 1 and + ((2000\ w (2000\ + 1 = 0. Hence (2000 = f(1) + wf(w) + w2f(w2) = 22000 + + + + w2)200° = 22000 + w(—w2)200° + w2(—w)200° = Thus 22000 + = 22000 — 1. + the desired value is 22000 — 1 3 Problem 52 Let x, y, z be positive real numbers such that x4 + y4 + z4 = Determine with proof the minimum value of 1— x8 + 52 For 0 < u < 1, let 1(u) = u(1 RY the AM-GM inequality, — 1— y8 u8). + 1 1— z8 Let A be a positive real number. A(f(u))8 =Au8(1 _u8)..(1 _u8) < [Au8 +8(1 _u8)]9 4. Solutions to Advanced 66 Setting A = in the above inequality yields 8 8(f(u))8 < (8) or It follows that 1 — x8 + 1 — y8 + = 1— x(1 — x8) + y(1 — y8) + z(1 — z8) > 8 with equality if and only if x=y = z 1 = Comment: This is a simple application of the result of problem 33 in the previous chapter. Problem 53 [Romania 1990] Find all real solutions to the equation 2X + 3X + 6X = x2. Solution 53 x < 0, the function f(x) = + + — x2 is increasing, so the equation 1(x) = 0 has the unique solution x = —1. Assume that there is a solution s 0. Then For — so s But then s and hence LsJ + 3S + 6S 3, 1. yields 2S 2LsJ = (1 + which in turn implies that GS > 4S = (23)2 I+ s, _____________ Solutions to Advanced Problems 28 + x 67 > s2, a contradiction. —1 is the only solution to the equation. problem 54 j4et {an}n1 be a sequence such that al = and 2 1 +— = for all n E N. Find an explicit formula for Solution 54 Solving the equation Note leads to x x 1 2 x that Therefore, 2+1) — and +i] 1 Problem 55 Let x, y, and z be positive real numbers. Prove that x + y y+f(y+z)(y+x) 1. 55 Note that .J(x+y)(x+z) 4. Solutions to Advanced Problems 68 In fact, squaring both sides of the above inequality yields x2+yz which is evident by the AM-GM inequality. Thus x x - — Likewise, v'V Y and z < Adding the last three inequalities leads to the desired result. Problem 56 Find, with proof, all nonzero polynomials f(z) such that f(z2) + f(z)f(z + 1) = 0. Solution 56 Let 1(z) = azm(z where m and n are non-negative integers — and g(z) = (z— zi)(z— 0 and ; 1, for i = 1,2,.. az2m(z . , + — = k. The given condition becomes — 1)m(z Zk), zi)(z2 — (z2 — Zk) —z2)...(z —zk) — .(z+1z1)(z+ 1—z2)(z+1—zk). a= and f is nonzero, so a = that is, m = n. Thus f is of the form Thus —a2, —1. Since z2 1, 1— z2 Then 22m = —zm(z — 1)mg(z). Dividing by Z2m(2 — + the last equation becomes g(z2) = g(z)g(z + 1). 0. ,lutions to Advanced Problems claim that g(z) one complex root r We 69 1. Suppose not; then clearly g must have at least 0. Now = g(r)g(r + = 0, = 0. g(r2) g(r4) g(r8) 1) = 0, and so on. Since g cannot have infinitely many roots, all its roots must have absolute value 1. Now, g((r — 1)2) = g(r soj(r—1)21 = — 1)g(r) = 0, 1. Clearly, if then 2 ' 2 But r2 is also a root of g, so the same should be true of r2: 7' 2 fi+vi 2 2 This is absurd. Hence, g cannot have any roots, and g(z) Therefore, the f(z) are all the polynomials of the form — for mEN, Problem 57 Letf: N —* N be a function such that f(n+ 1) > f(n') and f(f(n)) = 3n for all ri. Evaluate f(2001). 57, Alternative 1 We prove the following lemma. Lemma For ri=0,1,2,..., :1. 2. and 4. Solutions to Advanced Problems 70 We use induction. Proof For n = 0, note that f(1) 1, otherwise 3 = f(f(1)) = 1(1) = 1, which N, f(1) > 1. Since f(n + 1) > 1(n), is impossible. Since f N f is increasing. Thus 1 < 1(1) < f(f(1)) = 3 or 1(1) = 2. Hence I (2) = 1 (1 (1)) = 3. Suppose that for some positive integer ri 1, f and f (f(3n+1)) desired. This completes the induction. There are — 1 integers m such that 3fl — 1 integers m' such that = 3n+2, 0 as < in < 2 and there are . f 2 for 0 m 3n• 3fl +in, Therefore +m) =f(f (3fl +m)) = Hence 1(2001) = 1(2 . 36 + 543) = 3 (36 + 543) = 3816. Solution 57, Alternative 2 For integer n, let fl(3) = denote the base 3 representation of a1a2• Using similar inductions as in the first solution, we can prove that ifa1 =1. I = 1a2 if al = . 2. Since 2001(3) = 2202010, f(2001)(3) = 12020100 or 1 (2001) = 1• 32 +2 34 +2 36 + 37 = 3816. Advanced Problems 71 problem 58 [China 1999] Let F be the set of all polynomials f(x) with integers coefficients such f(x) = 1 has at least one integer root. the least integer greater than 1 for which For each integer k> 1, find F such that 1(x) = mk has exactly k distinct integer there exists an f E roots. Solution 58 has exactly Suppose that fk E F satisfies the condition that 1k (x) = k distinct integer roots, and let a be an integer such that fk(a) = 1. Let be the polynomial in F such that gk(X) for fk(x+a) all x. = fk(a) = 1, so the constant term of is 1. Now gk(x) = mk exactly k distinct integer roots r1, r2 rk, so we can write has Now gk(x) where qk(x) is — an integer polynomial. Note that r1r2• equals 1- mk. Since rnk > 1, mk = (x — r1)(x — r2) ... (x —. . . Tk divides the constant term of gk(x) 1 — 172k cannot 1— Now are — mk, which be 0, mk! rir2 - - distinct integers, and none of them isO. so rlr2..rkl hence mk Thi5 value of mk gk(x) is = Ik/21! + 1. attained by 1)(x+1)(x-2)(x+2) (x+ (_1)klk/2]) + Lk/2i! [k/2]! + 1. Thus, mk = Lk/2Th [k/2]! + 1. 4. Solutions to Advanced 72 Problem 59 Let x1 = 2 and = 1 1 Xi X2 1 1— 22 — xn + 1 2 Solution 59 Since x1 = 2 and — 1 = — 1), is increasing. Then — 1 0. _1 Hence — 1 1 — 1) — — 1 — 1 —1 — or —+—±...+—=1x2 x1 1 Thus it suffices to prove that, for m 1— 1 22 1 1 1 xn+1—1 N, 1 1 <1_ 22 or — (1) 1 We use induction to prove (1). For ri = 1, X2 = — x1 + 1 = 3 and (1) becomes 2 < 3 <4, which is true. Now suppose that (1) is true for some positive integer ii = k, i.e., 22k1 22k (2) — Then for ri = k + 1, Xk+2 — 1 the lower bound of (1) follows from = — 1) 22k—1 22k Solutions to Advanced Problems Since xk4-1 is 73 an integer, the lower bound of (2) implies that xk+1 22k and — 1, — 1 from which it follows that xk+2 — 1 = 22k Xk+1(Xk+1 — (22k 1) — i) 22k+1 desired. This finishes the induction and we are done. problem 60 [Iran 1997] R+ is a decreasing function such that for all Suppose that f : z,y E R+, f(x + y) + f(f(x) + 1(y)) = f(f(x + f(y)) + f(y + 1(x))). Prove that f(f(x)) = x. Solution 60 Setting y = x gives f(2x) + f(2f(x)) = f(2f(x + f(x))). Replacing x with f(x) yields f(2f(x)) + f(2f(f(x))) = 1(21(1(x) + f(f(x)))). Subtracting these two equations gives 1(21(1(x))) - f(2x) = f(2f(f(x) + f(f(x)))) - f(2f(x + 1(x))). If 1(1(x)) > x, the left hand side of this equation is negative, so f(f(x) +1(1(x))> f(x+ f(x)) and f(x)+f(f(x)) <x 1-f(x), contradiction A similar contradiction occurs if f(f(x)) <x. Thus f(f(x)) = x as desired. a Comment: In the original formulation I was meant to be a continous function. The solution above shows that this condition is not necessary. Solutions to Advanced 74 Problem 61 [Nordic Contest 1998] Find all functions f : Q —i Q such that f(x+y)+f(x -y)——2f(x)+2f(y) for all x,y EQ. Solution 61 The only such functions are 1(x) = kx2 for rational k. Any such function works, since f(x+y)+f(x—y)=k(x+y)2+k(x--y)2 =kx2+2kxy+ky2+kx2 —2kxy+ky2 = 2kx2 + 2ky2 = 21(x) + 21(y). Now suppose f is any function satisfying f(x+ y)+ f(x—y) =2f(x)+2f(y). Then letting x = y = 0 gives 21(0) = 41(0), so 1(0) = 0. We will prove by induction that f(nz) = n2f(z) for any positive integer ri and any rational number z. The claim holds for ri = 0 and ri = 1; let ri 2 and suppose the claim holds for n — and n — 2. Then letting x = (n — 1)z, y 1 z in the given equation we obtain f(riz) + f((n — 2)z) = f((n — 1)z + z) + f((n — 1)z — z) =2f((n—1)z)+2f(z) so f(nz) = 2f((ri — 1)z) + 2f(z) — f((ri — 2)z) = 2(n — 1)21(z) + 2f(z) — (n — 2)21(z) —(2n2—4n+2+2—n2+4n—4)f(z) = ri2f(z) and the claim holds by induction. Letting x = 0 in the given equation gives f(y) so n. f(—y) = 1(y) for + f(—y) = 2f(0) + 2f(y) all rational y; thus f(nz) = n2f(z) for all integers Solutions to Advanced Problems Now 75 let Ic = f(1): then for any rational number x = p/q, q2f(x) = f(qx) = 1(p) = p2f(1) = 5° Thus kp2 1(x) = kp2/q2 = kx2. the functions f(x) = kx2. k E Q. are the only solutions. Problem 62 [Korean Mathematics Competition 2000] Let <a < 1. Prove that the equation x3(x+1)=(x-i-a)(2x--a) has four distinct real solutions and find these solutions in explicit form. Solution 62 Look at the given equation as a quadratic equation in a: a2+ 3xa+2x2 —x3 —x4 =0. The discriminant of this equation is 9x2 — 8x2 + 4x3 + 4x4 = (x + 2x2)2. Thus a— The first choice a = whose solutions are —x —3x±(x+2x2) 2 + x2 yields the quadratic equation x2 — x — a = — (1±'/1+4a) 2 The second choice a = 2x — x2 yields the quadratic equation x2 + 2x + a = whose solutions are The inequalities Show that the four solutions are distinct. 0, 0. 4. Solutions to Advanced 76 Indeed reduces to < 3— v'l + 4a which is equivalent to 6V1 + 4a < 6 + 8a, or 3a < 4a2, which is evident. Problem 63 [Thurnament of Towns 1997] Let a, b, and c be positive real numbers such that abc = Prove that 1 1 1 <1. + + a+b+1 1. c+a+1 b+c+1 Solution 63, Alternative 1 Setting x = a + b, y = b + c and z = c + a, the inequality becomes x+1 + — y+l+ —<1, z+1 1 1 1 i.e. i.e. 1 1 y+l z+1 y+z+2 (y+1)(z+1) x x+1' < x x+1' i.e. xy+xz+2x+y+z+2 xyz+xy+xz+x, i.e. x+y+z+2 xyz, i.e. 2(a+b+c)+2 (a+b)(b+c)(c-4-a), i.e. 2(a + b + c) a2b + ab2 + b2c + be2 c2a + ca2. By the AM-GM inequality, (a2b-t-a2c+1) 3a. Advanced Problems Likewise, 77 (b2c+b2a+ 1) 3b (c2a+c2b+ 1) 3c. Therefore we only need to prove that 2(a+b+c)+3 3(a+b+c), i.e. 3 a+b + c, which is evident from AM-GM inequality and abc = 1. Solution 63, Alternative 2 b= Let a = C = Then a1b1c1 = 1. Note that 0, (ai which implies that a1b1(a1+bi). Therefore, 1 1 a+b+1 = 1 aibi(ai + b1) +a1b1c1 a1b1c1 aibi(ai + b1 + c1) Cl — a1 b1 a1+b1+c1 Adding the three inequalities yields the desired result. 4. Solutions to Advanced Problems 78 Problem 64 [AIME 1988] Find all functions f, defined on gers, the set of ordered pairs of positive inte.. satisfying the following properties: f(x,x)—x, f(x,y)=f(y.x). (x+y)f(x.y)=yf(x,x-4-y). Solution 64 We claim that f(x. y) = lcm(x. y). the least common multiple of x and y. It is clear that lcm(x, x) = x and lcm(x, y) = lcm(y, x). Note that lcm(x.y)= y gcd(x.y) and gcd(x.y) =gcd(x,x+y), where gcd (u. v) denotes the greatest common divisor of u (x+y)lcm(x,y) and v. Then = — x(x+y) gcd(x,x+y) = ylcm(x,x+y). Now we prove that there is only one function satisfying the conditions of the problem. For the sake of contradiction, assume that there is another function g(x, y) also satisfying the given conditions. Let S be the set of all pairs of positive integers (x, y) such that f(x, y) g(x, y), and let (in, ii) be such a pair with minimal sum m+ri. It is clear that in n, otherwise I (in, ii) = f(m. m) = in = g(in. in) = g(ni. n). By symmetry (f(x, y) = f(y, x)), we can assume that n — in > Note that nf(rn, n — in) = [in + (n — m)]f(rn. ii — in) = (m — ni)f (in, rn + (n — in)) = (71. — in)f(m., n) 0. Solutions to Advanced Problems or f(rn.ri—rn)= [,ikewiSe, g(rn, n — in) 79 n—rn •f(rnn). = ii . g(rn. n). Since f(rn. ii) g(m. n), f(rn. n — in) g(rn. n — in). Thus (in. n — rn) E S. But (in. n — in) has a smaller sum in + (n — in) = n. a contradiction. Therefore our assumption is wrong and f(x, y) lcm(x. y) is the only solution. problem 65 [Romania 1990] ii complex numbers zk. such that IzkI 1. k = 1.2 Prove that there exist c1.e2, . .. E (—1. 1} such that, for any in Consider . I + e2z2 + ii, + emzrnl 2. Solution 65 Call a finite sequence of complex numbers each with absolute value not exceeding 1 a green sequence. Call a green sequence happy if it has a friend sequence of is and — is, satisfying the condition of the problem. We will prove by induction on ii. that all green sequences are happy. For n = 2. this claim is obviously true. Suppose this claim is true when n equals some number in. For the case rn + 1. think of the zk as points in the complex plane. For each k. let ek be the line through the origin and the point corresponding to zk. Among the lines some two are within 600 of each other; suppose they are and with the leftover one being 4. The fact that and are within 600 of each other implies that there exists some number c9 {—1, 1} such that z' = + e9z9 has absolute value at most 1. the sequence z'. is a k-term green sequence. so, z4. z5 by the induction hypothesis, it must be happy: let c'. C4. C5 Ck+1 of ii = be its friend. Let Ca 1. Then the sequence comp'ete is the friend of Induction is now 4. Solutions to Advanced Problems 80 Problem 66 [ARML 1997] Find a triple of rational (a, b, c) such that Solution 66 Let x = —1 and y = Then y3 = and x = 2 Note that 1 =y3 —1 =(y— 1)(y2+y+ 1). and 2 — 3 3 x3 =y—l= = 3 (y+l)3 or (1) y+1 On the other hand, 3=y3+1=(y+1)(y2—y+1) from which it follows that 1 (2) y+l Combining L1 2 3 (1) and (2), we obtain x= Consequently, is a desired triple. (4 2 1 4 Solutions to Advanced Problems 81 problem 67 [Romania 1984) Find the minimum of + (X2 + + — are real numbers in the interval where Xl.x2 1). Solution 67 Since logs x is a decreasing function of x when 0 < a < (x — 1/2)2 0 implies x2 x — 1/4, we have = xk+1 and. since = — It follows that + 2 (logx2 + + (X3 + — — logx3 + logx2 + — + logxi 2n by the AM-GM inequality. Equalities hold if and only if Problem 68 [AIME 1984] Determine x2 + y2 22_12 z2 + w2 if +2232+2252 +22_72 =1, 42_ 12 + 42_32 + 42_52 + 42_72 = 62_12+62_32+ 82_12 z2 62 52+62 w2 72 1, 1, +8232+8252+8272=1. 4. Solutions to Advanced 82 Solution 68 The claim that the given system of equations is satisfied by and w2 is equivalent to claiming that z2 w2 t_12+t_32+i_52+t_72_l (1) satisfied by t = 4, 16. 36, and 64. Multiplying to clear fractions, we find that for all values of t for which it is defined (i.e., t 1, 9,25, and 49), (1) is equivalent to the polynomial equation is P(t) = 0, where P(t) = (t — 1)(t 9)(t — 25)(t — 49) 25)(t — 49) — y2(t — 1)(t — 25)(t — —x2(t — 9)(t —z2(i Since deg P(t) = 4, — — l)(t — 9)(t — 49) — w2(t — 1)(t — P(t) = 0 has exactly four zeros t = 9)(t 4, — — 49) 25). 16, 36, and 64, i.e., P(t) = (t 4)(t — — 16)(t — 36)(t — 64). Comparing the coefficients of t3 in the two expressions of P(t) yields I +9+25+49+x2+y2+z2+w2 =4+16+36+64, from which it follows that + y2 + z2 + w2 = 36. Problem 69 [Balkan 1997] Find all functions f : R IR such that f(xf(x) + 1(y)) = (f(x))2 + y for all x,y ER. Solution 69 Let f(0) = a. Setting x = 0 in the given condition yields I (f (y)) = a2 + for all y E R. Since the range of a2 +y consists of all real numbers, f must be surjective. 4 Solutions to Advanced Problems 83 there exists b C R such that f(b) = 0. getting x = b in the given condition yields Thus f(f(y)) = f(bf(b) + 1(y)) = for (1(b))2 +y= all y C IL It follows that, for all x, y (f(x))2 + y = f(xf(x) + f [1 (1 (x))f(x) + f(y)] = I [f(x)f(f (x)) + = f(f(x))2 + y = x2 + y, that is, yJ (f(x))2=x2. (1) It is clear that f(x) = x is a function satisfying the given condition. Suppose that f(x) x. Then there exists some nonzero real number c such that f(c) = —c. Setting x = cf(c) + f(y) in (1) yields [f(cf(c) + 1(y))]2 = [cf(c) + 1(y)]2 = for all y C R, and, setting x = c f—c2 + 1(y)]2, in the given condition yields f(cf(c)+f(y)) = (f(c))2+y =c2+y, for all y C IL that (f(y))2 = y2. It follows that f—c2 +1(y)]2 = (c2 +y)2, or f(y)=—y, for all y C R, a function which satisfies the given condition. Therefore the only functions to satisfy the given condition are 1(x) = x or 1(x) = —x, for x C R. Problem 70 The numbers 1000, ,2999 have been written on a board. Each time, one is allowed to erase two numbers, say, a and b, and replace them by the number min(a, b) After 1999 such operations, one obtains exactly one number c on the board. Prove that c < 1. 4. Solutions to Advanced 84 Solution 70 By symmetry, we may assume a b. Then 1. = 11 We have a 1 from which it follows that the sum of the reciprocals of all the numbers on the board is nondecreasing (i.e.. the sum is a monovariant). At the beginning this sum is s———-+—+...+—<-. 1000 1001 2999 — c 1 where 1 1 1/c is the sum at the end. Note that, for I 1 2000—k + 1 — 11 -> c — 1000 1 1 4000 4000 i\ 2999) as desired. Problem 71 [Bulgaria 1998] be real numbers, not all zero. Prove that the equation Let a1, a2 has at most one nonzero real root. Solution 71 Notice that = jij1jfJ /1 i, > 1, 999, k 2000+k — 20002—k2 > 20002 — Rearranging terms in S yields or c < 1 + a2x is concave. Hence 2998) 45olutions to Advanced Problems is 85 concave. Since f'(x) exists. there can be at most one point on the curve y = f(x) with derivative 0. Suppose there is more than one nonzero root. Since x = 0 is also a root, we have three real roots x1 < x2 < x3. Applying the Mean-Value theorem to 1(x) on intervals [xl.X2] and [x2,x3], we can find two distinct points on the curve with derivative 0, a contradiction. Therefore, our assumption is wrong and there can be at most one nonzero real root for the equation f(x) = n. Problem 72 [Turkey 1998] be the sequence of real numbers defined by a1 = Let = — 1. for n For t and how many distinct values of t do we have a1998 = 0? Solution 72, Alternative 1 Let 1(x) = 4x(1 — x). Observe that = {0. 1}, f1(1) = {1/2}. and 1]) = [0,1], = 2n_1 [0,1). = {x E R: Let = 0}; then = = We claim that for alln 1, For n c [0. 1]. 1 E = + 1. and 1. we have A1 = {x E R f(x) = 0} = I fO. 1}, and the claims hold. NOW 8o suppose n 1 and c [0. 1]. C [0. 1]. 1 e and + 1. Then 4. Solutions to Advanced 86 Since = 0 for all n 1, so I e f(O) = f(1) = 0, we have Now we have = = aE = {x:f(x)=l}I+ = 1+ I{x:f(x)=a}I > 2 aE aE[O,1) = = = 1+2(2n_1+1_1) Thus the claim holds by induction. Finally, a1998 = 0 if and only if I 1997(t) = values of t. 0, so there are 21996 + 1 such Solution 72, Alternative 2 As in the previous solution, observe that if f(x) E so if a1998 = 0 we must have t E [0. 1]. Observe that for any e f(sin2 = (1 — sin2 0. a2 = = = 4sin2 sin2 20. a1998 = a3 = a1ggg = sin2 219970. sin2 40 0 sin 219970 = 0 7Z. Thus the values of t which give a1998 = 0 are sin2 (kir/21997), k sin2 it follows that Therefore for some k e 1] then x e ir/2[ such that sin U = Now choose U e since [0, Z, giving 21996 + 1 such values of t. 0 = [0, 1], to Advanced Problems problem 73 [IMO 1997 short list] (a) Do there exist functions 1 : R —p R and g : f(g(x))=x2 R such that R such that and for all x e (b) Do there exist functions f R —i R and g R f(g(x)) = x2 g(f(x)) = and for all x E R? Solution 73 (a) The conditions imply that f(x3) = f(g(f(x))) = [1(x)]2, whence XE f—i, 0, 1} x3 = x ; : f(x) = ; f(x) E {O, 1}. Thus, there exist different a, b E f—I, 0, 1} such that 1(a) = 1(b). But then a3 = g(f(a)) = g(f(b)) = b3, a contradiction. Therefore, the desired functions f and p do not exist. (b) Let p(x) = if lxi 1 I 1 0 - In IxI if 0 < xi < I ifx=0. Note that g is even and al = bi whenever g(a) = are g(b); thus, we allowed to define f as an even function such that where y is such that p(+y) = x. 1(x) = We claim that the functions f, p described above satisfy the conditions of the problem. It is clear from the definition of f that f(p(x)) = x2. Now let y = Then g(y) = x and p(f(x)) = 9(y2) (y2)Ifl(Y2) = y4Ifl Y = = (yInY)4 ( = = 0 [9(y)]4 x4. if i if 0< y < 1 ify=0 4. Solutions to Advanced Problems 88 Problem 74 [Weichao Wu] LetO< a1 a2• Suppose <b1 that there exists 1 k n such that a, for 1 i k and for 1> k. b2 Prove that a1a2 Solution 74, Alternative 1 We define two new sequences. For i = 1.2 = / and n, Let —. = Then / , — ak = ak — — = —(a1 — or — — — = (ak — — Therefore nak = a2 + + + + ... + Applying the AM-GM inequality yields (bib2.bnafl* = from which the desired result follows. Solution 74, Alternative 2 We define two new sequences. For = 1.2 n, Let and Then (1) Note that, for cy(x — y)(y + c) 0 x+c x>yandc>0: y—y+c, — — x —>-———--. 45olutions to Advanced Problems Setting x= aj/bi Using y= and c = ak — the above inequality implies that for i = 1. 2, , n. Thus, . (1) and the AM-GM inequality yields ) F = +... + . ) or aia2 It is clear that the desired result follows from (2) and (3). Problem 75 Given eight non-zero real numbers a1, a2 a8, prove that at least one of the following six numbers: a1a3 + a2a4, aja5 + a2as, a1a7 + a2a8, a3a5 + a4a5, a3a7 + a4a8, + a6a8 is non-negative. Solution 75 [Moscow Olympiad 1978] First, we introduce some basic knowledge of vector operations. Let u = [a. b] and v = Im, nJ be two vectors. Define their dot product u v = am + bn. It is easy to check that (i) vv = m2+n2 = 1v12, that is, the dot product of vector with itself is the square of the magnitude of v and v• v 0 with equality if and only if v = [0.0]; (ii) u• v = v u; (iii) u (v + w) = u v + w. where w is a vector; (iv) (cu) v = c(u . v), where c is a scalar. . When vectors u and v are placed tail-by-tail at the origin 0. let A and .8 be the tips of u and v. respectively. Then = v — u. Let LAOB =0. Applying the law of cosines to triangle AOB yields = AB2 = 0A2-I-0B2-20A.OBcosO = juj2 + v]2 — 21u!JvJ cosO. 4. Solutions to Advanced Problem 90 It follows that (v — u) (v . — u) = uu+v v — 2IuIlvIcosO, or cosO= Consequently, if 0 0 IuHvI 900, u v 0. Consider vectors v1 = [ai, a2], v2 = [a3, a4], v3 = [a5, a6], and v4 [a7, as]. Note that the numbers a1a3+a2a4. a1a5+a2a6, a1a7+a2a8, a3a5+a4a6, a3a7 + a4a8, a5a7 + a6a8 are all the dot products of distinct vectors Vj and Since there are four vectors, when placed tail-by-tail at the origin, at least two of them form a non-obtuse angle, which in turn implies the desired result. Problem 76 [IMO 1996 short list] Let a, b and c be positive real numbers such that abc = Prove that ab ca bc 1. a5+b5+ab+b5+c5+bc+c5+a5+ca 1. Solution 76 We have a5 + b5 a2b2(a + b), because (a3 — b3)(a2 — b2) 0, with equality if and only if a = b. Hence ab ab a2b2(a+b)+ab 1 — ab(a+b)+1 — ab(a+b+c) abc C — a+b+c Likewise, bc b5 + c5 + bc a a+b+c 45olutions to Advanced Problems 91 and b + a5 + ca — a + b + c the last three inequalities leads to the desired result. Equality holds if and only if a = b = c = 1. c5 Comment: Please compare the solution to this problem with the second solution of problem 13 in this chapter. Problem 77 [Czech-Slovak match 1997] Find all functions f R —* R such that the equality f(f(x)+y) =f(x2 —y) +4f(x)y holds for all pairs of real numbers (x, y). Solution 77 Clearly. f(x) = x2 satisfies the functional equation. Now assume that there is a nonzero value a such that f(a) x2 —1(x) Let y = a2. in the functional equation to find that (f(x)+x2) (f(x)+x2) = 2f(x)(x2 — In 1(x) or 1(x) = x2. both cases. f(0) = 0. Setting x = a, it follows from above that either f(a) = 0 or 1(a) = 0 or f(a)=a2. = The latter is false, so f(a) Now, let x = 0 and then x = a 1(u) and Let in the = f(—y), functional equation to find that 1(y) = f(a2 — so 1(n) that 0. = f(—y) = f(a2 + y); is, the function is periodic with nonzero period a2. y= a2 in the original functional equation to obtain f(f(x)) = 1(1(x) + a2) = However, putting y = for all x. 0 in f(x2 — a2) + 4a2f(x) = f(x2) + 4a2f(x). the functional equation gives 1(1(x)) = 1(x2) 4. Solutions to Advanced Problems 92 Thus, 4a2f(x) = 0 for all x. Since a is nonzero, 1(x) = Therefore, either 1(x) = x2 or f(x) = 0 0 for all x. Problem 78 [Kvantj Solve the system of equations: 3x — y x2 + y2 x+ =3 x+3y =0. x2 + y2 Solution 78, Alternative 1 Multiplying the second equation by i and adding it to the first equation yields (3x—y)—(x+3y)i or i(x—yi) x2+y2 — x2+y2 Let z = x + yi. Then =3, !_ x—yi z — x2 + y2 Thus the last equation becomes z+ 3—i z = or z2—3z+(3—i) =0. Hence 3±v'—34-4i — 3±(1+2i) 2 — 2 — that is, (x,y) = (2,1) or (x,y) = (1,—i). Solution 78, Alternative 2 Multiplying the first equation by y, the second by x, and adding up yields 2xy + or 2xy — 1 = 3y. (3x — y)y (x + 3y)x x2 + y2 It follows that y — 0 and 3y + I = 45olutions to Advanced Problems this into the second equation of the given system gives [(3y+ 1)2 + (3Y+ 1) — = o. or 4y4 — 3y2 it follows that y2 — 1 = 0. 1 and that the solutions to the system are (2, 1) and (1,—i). problem 79 [China 1995] Mr. Fat and Mr. Taf play a game with a polynomial of degree at least 4: + ... +_x + i. They fill in real numbers to empty spaces in turn. If the resulting polynomial has no real root. Mr. Fat wins; otherwise, Mr. Taf wins. If Mr. Fat goes first, who has a winning strategy? Solution 79 Mr. Taf has a winning strategy. We say a blank space is odd (even) if it is the coefficient of an odd (even) power of x. First Mr. Taf will fill in arbitrary real numbers into one of the remaining even spaces, if there are any. Since there are only n Space left after 2n — 3 — 1 even spaces, there will be at least one odd plays, that is, the given polynomial becomes p(x) = q(x) where +_x2t_1, s and 2t — I are distinct positive integers and q(x) is a fixed Polynomial. We claim that there is a real number a such that p(x) = q(x) + Will +_x2t1 always have a real root regardless of the coefficient of Then Mr. Taf can simply fill in a in front of x8 and win the game. 4. Solutions to Advanced Now we prove our claim. Let b be the coefficient of x2t_1 in p(x). Note that + p(l) + 2s_2t+la + b) + [q(—1) + (—1)8a — = Since s = + q(_1)) + 2t — 1 2s—2t+1 + (_1)8 + o. Thus 1 a= is + q(—1) — 2s2t+1 + (_1)8 well defined such that a is independent of b and +p(—l) = 0. It follows that either p(—l) = p(2) = 0 or p(—l.) and p(2) have different signs, which implies that there is a real root of p(x) in between —1 and 2. In either case, p(x) has a real root regardless of the coefficient of as claimed. Our proof is thus complete. Problem 80 [IMO 1997 short list} Find all positive integers k for which the following statement is true: F(x) is a polynomial with integer coefficients satisfying the condition for if c=0,1 then F(O) = F(1) = ... = F(k + 1). Solution 80 The statement is true if and only if /c 4. We start by proving that it does hold for each Ic 4. Consider any polynomial F(x) with integer coefficients satisfying the inequality 0 F(c) Ic for each c e {0. 1 k + 1}. Note first that F(k + 1) = F(0), since F(k + 1) —F(0) is a multiple of Ic + 1 not exceeding Ic in absolute value. Hence F(x) — F(0) = x(x — Ic — 1)G(x). 45olutions to Advanced Problems G(x) is a polynomial with integer coefficients. consequently, where k jF(c) — F(O)j = c(k + I — c)IC(c)I for each c E 2 k}. The equality c(k + 1 — c) > k holds for each cC {2, 3 equivalent to (c — 1)(k — c) > 0. k — 1}, as it is that the set f2. 3 k — 1} is not empty if k 3, and for any c in this set. (1) implies that < 1. Since C(c) is an integer, C(c) 0. Note Thus F(x)—F(0)=x(x—2)(x—3)''(x—k+1)(x—k—1)H(x). (2) where H(x) is a polynomial with integer coefficients. To complete the proof of our claim, it remains to show that 11(1) = H(k) = 0. Note that for c = 1 and c = k, (2) implies that Ic IF(c) — F(0)I = (Ic — 2)! . Ic IH(c)I. Fork4,(k—2)!>1. Hence H(c) = 0. We established that the statement in the question holds for any k 4. But the proof also provides information for the smaller values of Ic as well. More exactly, if F(x) satisfies the given condition then 0 and k + 1 are roots of F(x) and F(0) for any Ic and if Ic 3 then 2 must also be a root of F(x) — F(0). Taking this into account, it is not hard to find the following counterexamples: F(x)=x(2—x) fork=1. for Ic = 2. F(x) = x(3 — x) F(x)=x(4—x)(x—2)2 for k=3. 4. Solutions to Advanced Problems 96 Problem 81 [Korean Mathematics Competition 2001J The Fibonacci sequence is given by (n.EN). Prove that r'3 i'3 L 2n+2 £ 2n—2 9 for all n 2. Solution 81 Note that = — = — = — whence for =0 — — (1) all n 2. Setting a = and c = b= in the algebraic identity —ab—bc—ca) gives rv,n3 htf2n — r,3 n3 — n — — — Applying (1) twice gives = — — — — — — 2 — — — — The desired result follows from tlr2n+2r2nr2n_2 r'3 — r' — — i r' r' r'2 — — r2n)\ — 2 4. Solutions to Advanced Problems 97 problem 82 [Romania 1998] Find all functions u : R —* R for which there exists a strictly monotonic function f : R —* R such that f(x + = f(x)u(y) + f(y) y) for all x,y ER. Solution 82 The solutions are u(x) = ax. a E R±. To see that these work, take f(x) = x for a = 1. 1, take 1(x) =a —1; then f(x + y) = —1 = (ax — = 1 + f(y) for all x, y E R. Now suppose u R —* R are functions for which f is strictly R, f R monotonic and f(x + y) = f(x)u(y) + f(y) for all x, y E R. We must show that u is of the form u(x) = ax for some a E R+. First, letting y = 0, we obtain f(x) = f(x)u(0) + f(0) for all x E R. 1 would imply f(x) = f(O)/(1 — u(O)) for all x, which Thus, n(O) would contradict the fact that f is strictly monotonic, so we must have u(0) = 1 and f(0) = 0. Then f(x) 0 for all x 0. Next, we have f(x)u(y) + f(y) = f(x + y) = f(x) + f(y)u(x), or f(x)(u(y) — 1) = f(y)(u(x) — 1) for all x,y ER. That is, for all xy u(x) — I — f(x) — u(y) — 1 1(y) 0. It follows that there exists C E R such that u(x) — I f(x) for all x = 0. Thus, u(x) = 1+Cf(x) for x also holds for x = 0. 0: sinceu(0) = 1, f(0) = 0, this equation 4. Solutions to Advanced Problem 98 If C = 0, then u(x) = 1 for all x, and we are done. Otherwise, observe u(x+y) = 1+Cf(x+y) + Cf(x)u(y) + Cf(y) = u(y)+Cf(th)u(y) = 1 = u(x)u(y) for all x,y cR. Thus u(nx) = u(x)°° for all n Z, x R. Since u(x) = 1 + Cf(x) for all x, u is strictly monotonic, and u(—x) = 1/u(x) for all x, so u(x) > Let a = u(1) > 0; for all x as u(O) = 1. then u(n) = a°° for all n N, and 0 u(p/q) = (u(p))lk = for allpe Z, q E N, so u(x) = ax for allxE Q. monotonic and the rationals are dense in for all x E R. Thus all solutions are of the form u(x) = ax, a E Since u is we have u(x) = ax Problem 83 [China 1986] be complex numbers such that Let z1 Z2, , , Prove that there exists a subset S of {zi, Z2,... , such that z€S Solution 83, Alternative 1 Let £2, and £3 be three rays from origin that form angles of 60°, 180°, and 300°, respectively, with the positive (here £4 = For i = 1,2,3, let 7Z, denote the region between and Then including the ray 1= jZkj+ > ZkH !ZkI ZkEl?..3 By the Pigeonhole Principle, at least one of the above sums is not less than 1/3. 4. Solutions to Advanced Problems (otherwise, we apply a rotation, which does not effect the it's magnitude of a complex number), Let zk = xk + 1Yk. Then for Zk e 7Z3, Say XkI Xk ZkI/2. > as desired. Solution 83; Alternative 2 We prove a stronger statement: there is subset S of {zi, z2, .. . that , z,.,} such zES For I k n, let Zk 1 Xk + iyk. Then = = XkO Xk<O > yk<O By the Pigeonhole Principle, at least one of the above sums is not less than 1/4. By symmetry, we may assume that Xk>O XkO Consequently, XkO Comment: Using advanced mathematics, the lower bound can be further improved to 1/ir. Problem 84 [Czech-Slovak Match 1998] A polynomial P(x) of degree n 5 with integer coefficients and n distinct integer roots is given. Find all integer roots of P(P(x)) given that 0 is a root of P(x). 4. Solutions to Advanced Problems 100 Solution 84 The roots of P(x) are clearly integer roots of P(P(x)); we claim there are no other integer roots. We prove our claim by contradiction. Suppose, on the contrary, that P(P(k)) = 0 for some integer k such that P(k) 0. Let P(x) =a(x—ri)(x— r2)(x—r3).. .(x— rn), where a.rl,r2,...,rfl are integers, Since P(k) Since the we must have 1k — 1 for all i. are all distinct, at most two of 1k — r2j. k — 0, equal r31, 1k — 1, so Ia(k—r2).(k—rn_i)I IP(k)l — Also note that P(k) = for some i0, so P(k)! Now we consider the following two cases: and 1. Then P(k)I rn!. 1k! — > rn!, a contradic- rn)! tion. 2. 1k! a < rn!, that is, b. For xe [a, 1 1k! b}, the — 1. Let a. b, c be real numbers, function f(x) =x(c—x) reaches its minimum value at an endpoint x = a or x = b, or at both endpoints. Thus k(k — rn)I = k!!rn — kI !k!(!Tn! — — 1. 1k!) It follows that rn! P(k)! — 1), — which implies that Tn! K 2. Since n 5, this is only possible if P(x) = (x + 2)(x + 1)x(x — 1)(x But then it is impossible to have k diction. — 2). and 1k! rn!, a contra- 4 Solutions to Advanced Problems 101 our assumption was incorrect, and the integer roots of P(P(x)) are exactlY all the integer roots of P(x). Thus problem 85 [Belarus 1999} and Two real sequences x1, x2 are defined in the following Y2 way. and yn — 1 + + for all n 1. Prove that 2 < XnYn <3 for all n> 1. Solution 85, Alternative 1 Let = 1 and note that the recursion for is equivalent to + Zn+1 = Zn + = x1; since the and Also note that Z2 = recursion, this means that Zn = Xn_1 for all n> 1. satisfy the same Thus, XnYn = Xn —= 2n Xn —. Xn_1 Note that > Thus 2Xn_i and XnYn > 2, which is the lower bound of the desired inequality. Since Xn5 are increasing for n> 1, we have = 3> which implies that 2Xn_i > Thus 3Xn_i > Xn. which leads to the upper bound of the desired inequality. Solution 85, Alternative 2 Setting = cot 0,, for 0 < 0,, <900 yields = cot + + cot2 = cot + = cot 4. Solutions to Advanced Problen 102 = Since 300 30°, we have in general Similar calculation shows that tan 2 2 tan 1 tan 0. tan2 And since for n> 1 is positive and XnYn > 2. < 30°. we have < tan2 so that XnYn we also hare <3. Comment: From the closed forms for and in the second solution, we can see the relationship yn = used 1 xfl—l in the first solution. Problem 86 [China 1995] For a polynomial P(x), define the difference of P(x) on the interval 4 b) ({a,b), (a,b), (a,bJ) as P(b) — P(a). Prove that it is possible to dissect the interval 9. 1] into a finite number of intervals and color them red and blue alternately such that, for every the total difference equal to that of P(x) on blue intervals. What about cubic polynomials? quadratic polynomial P(x), of P(x) on red intervals is Solution 86 For an interval i, let denote the difference of polynomial P on i. For a positive real number c and a set S ç R, let S + c denote the set obtained by shifting S in the positive direction by c. We prove a more general result. Lemma Let £ be a positive real number, and let k be a positive integer. It is always possible to dissect interval Ik [0. into a finite number of such that, for every polynomial P(x) with degP k, the total difference of P(x) on the red intervals is equal to that on the blue intervals. intervals and color them red and blue alternatively 45olutions to Advanced Problems 103 proof We induct on k. For k = 1, we can just use intervals and (t.2t]. It is easy to see that a linear or constant polynomial has the same difference on the two intervals. Suppose that the statement is true for /c = n, where n is a positive integer; that is. there exists a set of red disjoint intervals and a set fl of blue disjoint intervals such that U = and, = @. for any polynomials P(x) with deg P n. the total differences of P on is equal to that of P on consider polynomial f(x) with deg f n + 1. Define g(x) = f(x + Then deg h and h(x) = f(x) — g(x). n. By the induction hypothesis, = or Arf+ It follows that Arf, where 11-n+1 and — — D L)ii(D T = U + )flD\ and both contain the number that number may be removed from one of them.) It is clear that and form a dissection of 'n+l and, for any (If Polynomial f with degf equal to that of f on n + 1, the total difference of f on The only possible trouble left is that the colors in be alternating (which can happen at the end of the is might not and the beginning But note that if intervals i1 = [a1, b1 J and i2 = [b1, c1 } are in the same Color, then + Wherej3 = [ai,ci]. = 4. Solutions to Advanced Problems 104 we can simply put consecutive same color intervals into one bigger interval of the same color. Thus, there exists a dissection Thus. in = such that, for every polynomial f(x) with deg f n + 1. This completes the induction and the proof of the lemma. Setting first £ = and then £ = 0 in the lemma, it is clear that the answer to each of the given questions is Problem 87 [USSR 1990] Given a cubic equation S3 + _X2 + _X + — = 0, Mr. Fat and Mr. Taf are playing the following game. In one move, Mr. Fat chooses a real number ana Mr. Taf puts it in one of the empty spaces. After three moves the game is over. Mr. Fat wins the game if the final equation has three distinct integer roots. Who has a winning strategy? Solution 87 Mr. Fat has a winning strategy. Let the polynomial be x3 + ax2 + bx + c. Mr. Fat can pick 0 first. We consider the following cases: (a) Mr. Taf chooses a = 0, yielding the polynomial equation x3 + bx + c = 0. Mr. Fat then picks the number —(inrip)2, where in. n, and p are three positive integers such that m2 + n2 = If Mr. Taf chooses b = —(n2rip)2, then Mr. Fat will choose c = 0. The given polynomial becomes x(x — n2np)(x + rnnp). 4golutions to Advanced Problems If Mr. Taf chooses c = —(rnnp)2. then Mr. Fat will choose fl2p2 b = ?TL2fl2 — p27772. — The given polynomial becomes (x + rn2)(x + n2)(x — p2). (b) Mr. Taf chooses b = 0. yielding the equation x3 + ax2 + c = 0. Mr. Fat then picks the number m2(m + 1)2(rn2 -+ m + where rn is an integer greater than 1. If Mr. Taf chooses a = rn2(m + 1)2(m2 + m + then Mr. Fat can choose c = —m8(in + 1)8(in2 + rn + 1)6. The polynomial becomes (x rnp)[x (rn 1)pJ[x + m(rn + 1)pJ, where p = m2(n2 + 1)2(rn2 + rn + 1)2. If Mr. Taf chooses c=m2(m+ 1)2(m2 +m+ j)3, then Mr. Fat can choose a= —(m2 +m+ 1)2. The polynomial becomes (x+mq)[x—(rn+1)qffx—m(m+ where q = in2 + in + 1. 4. Solutions to Advanced Problems 106 (c) Mr. Taf chooses c = 0. Then the problem reduces to problem 40 of the previous chapter. Mr. Fat needs only to pick two integers a and b such that ab(a— 1)(b—1) and a+b= —1. The polynomial becomes either x(x — 1)(x — a) or x(x — 1)(x — b). Our proof is complete. Below is an example of what Mr. Fat and Mr. Taf could do: F T F 0 a —3600 b 4.9.73 T F b 0 —481 c Roots —60,0.60 .38.76 a —16, —9,25 —8.27.49, 4. 27 . 49, 89. c " " 2 " —49 a —3 —14, 21,42 —3,0,1 b —3 0,1.2 Problem 88 [Romania 1996] Let n> 2 be an integer and let f : R2 any regular n-gon A1A2. . 49 c R be a function such that for . f is the zero function. Solution 88 We identify R2 with the complex plane and let c = Then the condition is that for any z C C and any positive real t, In particular, for each of k 1,. .. , n, we obtain 4. Solutions to Advanced Problems 107 we have gumming over ,n=1 k=1 Form = n the inner sum is nf(z); for other m, the inner sum again runs over a regular polygon, hence is 0. Thus f(z) = 0 for all z E C. Problem 89 [IMO 1997 short list] Let p be a prime number and let f(s) be a polynomial of degree d with integer coefficients such that: (i) 1(0) = 0, f(1) = 1; (ii) for every positive integer n, the remainder upon division of 1(n) by p is either 0 or 1. Prove that dp— 1. Solution 89, Alternative 1 For the sake of the contradiction, assume that d p — 2. Then by Lagrange's interpolation formula the polynomial f(s) is determined by its values at 0, 1, 1 (x) . . ., p — 2; that is, — — k=O — p-2 — Setting x = p k)5 k=O — 1 f(p — (—1) .. (k 1 — xk (x k!(_1)P-k p +2) (x P+2) (p — k —2)! gives 1) = = >f(k) — — — k) (_1)Pkk! p—2 (modp). It follows that S(f) := f(o) + f(1) + ... + f(p -1) 0 (mod p) (1) 4. Solutions to Advanced Problems 108 other hand, (ii) implies that S(f) j (mod p), where j the number of those Ic E 1 1 p — 1} for which 1(k) (mod p) But (i) implies that 1 j < p — 1. So S(f) 0 (mod p), which contradicts (1). Thus our original assumption was wrong. and our proof is complete. On the Solution 89, Alternative 2 Again, we approach the problem indirectly. Assume that d p — 2, and let 1(x) = + + a1x + a0. Then p—i S(f) = p—i p—2 p—2 1(k) = p—2 = i=0 k=O i=0 k=0 p—i = k=O i=0 p—i where = (modp)foralli=0,1 p—2. We use strong induction on I to prove our claim. The statement is true for i 0 as So = p. Now suppose that So S1 0 (mod p) for some 1 i •.. p—2. Note that p—i p p—I k=Oj=0 (I + j—0 (mod p) Since 0 < i + 1 <p, it follows that 0 (mod p). This completes the induction and the proof of the claim. Therefore, S(f) 0 (mod p). = The rest is the same as in the first solution. 4Solutions to Advanced Problems 109 problem 90 Let n be a given positive integer. with a0 = Consider the sequence a0, a1. ak = ak_I for k = 1, 2, Prove that , and +— n 1 n. 1 < 1. 1— — Ti Solution 90, Alternative 1 We prove a stronger statement: For k = 1. 2 71 (1) We use induction to prove both inequalities. We first prove the upper bound, For k = 1, it is easy to check that 1 1 2n+1 n 2 4n 4n 2n—i a1 = — + — = Suppose that < 2n— k' for some positive integer k <n. Then ak = —(n-f-ak) < 271_k(Ti+271_k) — n(2n—k+1) — (2n—k)2 < 2n—k—1' (2n —k+ 1)(2n— k—i) = (2n—k)2 —1< (2n— k)2 Thus our induction step is complete. In particular, for k = n afl=ak+1< n 2n — (n — 1) — I =1, — 1, 4. Solutions to Advanced Problems 110 as desired. Now we prove the upper bound. For k = = 2n+1 1, it is easy to check that n+1 4n Suppose that n+1 2ri — k + 2' ak> for some positive integer k < n. Then ak+l — ak + (n -I- 1)2 ri + 1 n > 2n_k+2+n(2n_k+2)2 It follows that n+1 n+1 ak+i_2k+l (ri.l-1)2 (2n_k+l)(2n_k+2)+n(2n_k+2)2 n+1 "ri+l — n+1 /1 — 2n—k+2)2 2n—k+2\ = This complete the induction step. In particular, for n n+1 2n—(n—1)+1 as 1 2n_k+1) >0. = k — 1, 72+1 n+2 n+2 ri desired. Solution 90, Alternative 2 Rewriting the given condition as Ti 1 ak 1 a ak we obtain rt+ak_1 It is clear that aks are increasing. Thus I 2 1 4. Solutions to Advanced Problems 111 Thus (2) implies that 1 ———<— n ak_i 1 1 ak for k = 1, 2, . .. , fl Telescoping summation gives 1 1 ———<1 a0 or 1 —> that is, 1 ——1 = 2—1 = a0 <1, which gives the desired upper bound. = < 1, and, since aks are increasing, Since 1, < 1 for < k=1,2,...,n. Then (2) implies 1 1 1 ak n+ak_1 > n+1 fork=1,2,...,ri. Telescoping sum gives 1 1 —--—> a0 n+1 or 1<1 n+ln+1 ao that is, n+2 72 n+1 =1— 1 >1——, n n+2 n-f-2 which is the desired lower bound. Problem 91 [IMO 1996 short list] Let a1, a2,.. . , be nonnegative real numbers. not all zero. (a) Prove that — positive real root R. (b) Let A = — and B = Prove that AA RB. .. — — jaj. = 0 has precisely one 4. Solutions to Advanced Problem 112 Solution 91 (a) Consider the function a1 a2 Note that f decreases from oo to 0 as x increases from 0 to oc. Hence there is'a unique real number R such that f(R) = 1, that is, there exists a unique positive real root R of the given polynomial. (b) Let c3 = ad/A. Then c3s are non-negative and c2 = 1. Since — ln x is a convex function on the interval (0, oo), by Jensen's inequality, Ecj =-In(f(R))=0. —ln ft follows that c3(—lnA+jlnR) 0 j=1 or c3 = a3 /A, we obtain the desired inequality. Comment: Please compare the solution of (a) with that of the problem 15 in the last chapter. Problem 92 Prove that there exists a polynomial P(x, y) with real coefficients such that P(x, y) 0 for all real numbers x and y, which cannot be written as the sum of squares of polynomials with real coefficients. Solution 92 We claim that P(x,y) = (x2 +y2 — 1)x2y2 + is a polynomial satisfying the given conditions. First we prove that P(x, y) 0 for all real numbers x and y. 4. Solutions to Advanced Problems 113 If x2 + y2 — I 0, then it is clear that P(x,y) > 0; if x2 + y2 —1 <0, then applying the AM-GM inequality gives or (x2 + y2 — l)x2y2 It follows that P(x,y) 0. We are left to prove that P(x, y) cannot be written as the sum of squares of polynomials with real coefficients. For the sake of contradiction, assume that P(x,y) = Since degP = 3. 6, Thus = + + C2y2 + y)2, of terms x6 Comparing the coefficients, in P(x, y) and and y6 gives =0, orA' =D2=Oforalli. Then, comparing those of x4 and y4 gives = = Next, comparing those of x2 and y2 gives = or H, = 0, for all Thus, = + I2y + + F,.xy + 4. Solutions to Advanced Problems 114 But, finally, comparing the coefficients of the term x2y2, we have = -1, which is impossible for real numbers Thus our assumption is wrong, and our proof is complete. Problem 93 [IMO 1996 short list] For each positive integer n, show that there exists a positive integer k such that k = f(x)(x + 1)2n + g with integer coefficients, and find the smallest such /c as a function of n. Solution 93 First we show that such a k exists. Note that x + 1 divides 1 — Then for some polynomial a(s) with integer coefficients, we have (1 + x)a(x) = 1 — = 2— (1 + or 2=(1+x)a(x)+(1 +x272). Raising both sides to the (2n)th power, we obtain = (1 + + (1 + where b(s) is a polynomial with integer coefficients. This shows that a k satisfying the condition of the problem exists. Let /c0 be the minimum such k. Let 2n = 2r . q, where r is a positive integer and q is an odd integer. = We claim that First we prove that 2q divides k0. Let t = (5t + 1)Q(x), where Q(x) = 5t(q_l) The roots of — Note that + ... — + + 1. + 1 are ((2m—1)ir\ . . f(2m—1)ir\ ), m=1,2,...,t; 1 Solutions to Advanced Problems that is, Let f(s) 115 R(x)=xt+l=(x_wi)(x_w2)..(x_wt). and g(x) be polynomials with integer k0 coefficients — f(x)(x+1)272+g(x)(x272+1) = f(x)(x + 1)2n + g(x)Q(x)(xt + 1). such that It follows that f(Wm)(Wm+1)Th=ko, (1) Since r is positive, t is even. So 2=R(—1) = +w2)•(1+wt). is a symmetric polynomial in Since .. , with integer coefficients, it can be expressed as a polynomial with integer coefficients in the elementary symmetric functions in w1, w2,. and .. , therefore F = f(wl)f(w2)• . is an integer. Taking the product over m = 1,2,... , t, (1) gives 2272F = ku or = It follows that 2q divides k0. It now suffices to prove that k0 Note that Q(—1) = 1. It follows that Q(x) =(x+1)c(x)+1, where is a polynomial with integer coefficients. c(s) Hence (x + 1)2n = Q(x)d(x) + 1, for some polynomial d(x) with integer coefficients. Also observe that, for any fixed m, Thus (1 +(iJm)(1 and writing (1 +w2t_1) = R(—1) = 2, (2) 4. Solutions to Advanced Problems 116 we find that for some polynomial h(x), independent of m. with integer such that coefficients (1 + w)th(w) = 2. But then (x + 1)h(x) — 2 is divisible by xt + 1 and hence we can write (x + 1)h(x) = 2+ (xt + 1)u(x), for some polynomial u(x) with integer coefficients. Raising both sides to the power q we obtain = 2q + (xt + 1)v(x), (x + (3) where v(x) is a polynomial with integer coefficients. Using (2) and (3) we obtain (x + + 1)v(x) = Q(x)d(x)(xt + 1)v(x) + (xt + 1)v(x) = Q(x)d(x)(xt -I- 1)v(x) + (x + — 2q, that is, 2q where (x) = fi(x)(x + g1(x) + + 1), and 91(x) are polynomials with integer coefficients. Hence k0 < 2q, as desired. Our proof is thus complete. Problem 94 [USAMO 1998 proposal, Kiran Kedlaya] Let x be a positive real number. (a) Prove that (n—i)! _! (b) Prove that (n—i)! 1 4. Solutions to Advanced Problems 117 Solution 94 We use infinite telescoping sums to solve the problem. (a) Equivalently, we have to show that 1 1 that n!x (n—i)! — n! and this telescoping summation yields the desired result. (b) Let (n-i)! f( )X Then, by (a), 1(x) In particular, 1(x) write f as converges to 0 as x approaches oo, so we can an infinite telescoping series = +k -1) - f(x + k)]. On the other hand, the result in (a) gives f(x-1)-f(x) = — — (n—i)! x1(x+1)•(x+n) (1) 4. Solutions to Advanced Problems 118 Substituting the la.st equation to (1) gives as desired. Problem 95 [Romania 1996] Let n 3 be an integer, and let XcS={1,2,,,.,n3} be a set of 3n2 elements. Prove that one can find nine distinct numbers such that the system aix+biy+ciz = a2x+b2y+c2z = a3x+b3y+c3z = (i = 1,2,3) in X 0 0 0 has a solution (xo, Yo, zo) in nonzero integers. Solution 95 Label the elements of X in increasing order Xi << and put X1 = {x1,... X2 X3 Define = = the function f : X1 x X2 x X3 f(a, b, c) = (b S — x S as follows: a, c — b). The domain of f contains n6 elements, The range of f, on the other hand, is contained in the subset of S x S of pairs whose sum is at most n3, a set of cardinality 1 6 By the Pigeonhole Principle, some three triples (a2, map to the same pair, in which case x = — = ci is a solution in nonzero integers. — cj) (i = 1,2,3) a1,z = a1 — bi 4. Solutions to Advanced Problems Note ai 119 that cannot equal since X1 and X2 are disjoint, and that a2 implies that the triples (ai,bi,c1) and (a2,b2,c2) are identical, a contradiction. Hence the nine numbers chosen are indeed distinct. problem 96 [Xuanguo Huang] Let n 3 be an integer and let x1, x2, , be positive real numbers. Suppose that = 1. Prove that (1 1 Solution 96 By symmetry, we may assume that x1 x2 following lemma. Lemma 1 Prooft + — + x3 1 Since n 3, and, since = 1, 1 we have — 1 1 or > It follows that x2x3> 1. Thus — — 1+x2 1+x3 (1+x1)(1+x3) — - (1+x2)(1+x3) — 0, We have the 4. Solutions to Advanced Problems 120 as o desired. By the lemma, we have > 1+xi — 1+x2 — — and, since it follows by the Chebyshev Inequality 1 —1 (1 — By the Cauchy-Schwartz Inequality, we have 2 or (2) Multiplying by on both sides of (2) and applying (1) gives which in turn implies the desired inequality. 4. Solutions to Advanced Problems 121 problem 97 be distinct real numbers. Define the polynomials Let X1,X2,. . . P(x) = /1 and Q(x)=P(x)( \X—Xj + 1 X—X2 +..,+ Yn—1 be the roots of Q. Let Y1,Y2 Show that minlx,,—x31 <minly2—y71. Solution 97 By symmetry, we may assume that d= minly, —Yjl= Let 8k = — Xk, for k = 1.2,..., n. By symmetry, we may also assume that .s1 < < i.e., x1 > For the sake of contradiction, assume that = minx, — d minlx2 — = mins3 — (1) i(j Since P has no double roots, it shares none with Q. Then 1 (\Y,—X1 + 1 yj—X2 + 1 = Q(Yi) 0, or 1 YiX1 + 1 1 Y%—X2 =0. In particular, setting i = 1 and i = 2 gives (2) We claim that there is a k such that sk(sk + d) <0, otherwise, we have 1 1 Sk+d 8k 4. Solutions to Advanced Problems 122 for. all k, which in turn implies that which contradicts (2). < 0 < Sk+d. Let j be the number of ks such that sk(sk+d) < 0, that is, Sk the same sign. In have d and + A simple but critical fact is that 8k fact, suppose that then > 0 if and only if /c > i + 1, that is Sk + d> 0. Then From (1), we obtain Sk + d Sk+3, and, since sk + d and 3k+3 have the same sign, we obtain 1 Sk for all k = . ,n >1 + d — 8k+j — j. Therefore, or (3) Also note that E!<o<E 1• + k=1 d Sk Adding (3) and (4) yields Ti fl which contradicts (2). Thus our assumption is wrong and our proof is 4. Solutions to Advanced Problems 123 Problem 98 [Romania 1998] Show that for any positive integer n, the polynomial f(x)=(x2+x)2"+l cannot be written as the product of two non-constant polynomials with integer coefficients. Solution 98 Note that 1(x) = g(h(x)), where h(x) = x2 + x and 9(y) = y2" + 1. Since and is even for criterion. 1 k — 1, g is irreducible, by Eisenstein's Now let p be a non-constant factor of f, and let r be a root of p. Then g(h(r)) = f(r) = 0, so .s h(r) is a root of g. Since s = r2 -f- r C Q(r), we have Q(s) C Q(r), so deg p deg(Q(r)/Q) deg(Q(s)/Q) = deg g= Thus every factor of f has degree at least T'. Therefore, if f is reducible, we can write f(x) = A(x)B(x) where A and B have degree 5 Since have +5 2 +x+1) 2" (mod2). + x + 1 is irreducible in Z2[x], by unique factorization we must A(s) B(s) + x + 1)2"' + + 1 (mod 2). Thus, if we write A(s) = B(s) = + ao, + + .. + b0, b0 are odd and all the other coefficients then ao, are even. Since f is monic, we may assume without loss of generality 4. Solutions to Advanced Problems 124 = 1; also, a0bo = f(O) = = that no real roots, so a0 = b0 = 1. 1. but a0 > 0, > 0 as f has Therefore, ([x2n+2"1] + [x2"1])(g(x)h(x)) + ( i=2"1 + + EQ as But + + (mod 4) is even. + [x2"'])(f(x)) = is odd by Lucas's theorem, so and ([x2"+2"'] + [x2"')) (1(x)) 2 (mod 4), a contradiction. Hence f is irreducible. Problem 99 [Iran 1998] Let Ii, 12,13 R R be functions such that aifi + a212 + a3f3 is monotonic for all a1, a2, a3 E Prove that there exist Ci, c2, c3 c R, not all zero, such that cifi(x) + c2f2(x) + c3f3(x) = 0 for all x E R. Solution 99, Alternative 1 We establish the following lemma. Lemma: Let f, g R —p IR be functions such that f is nonconstant and af + bg is monotonic for all a, b c R. Then there exists c R such that g — cf is a constant function. Proofi Let s, t be two real numbers such that f(s) 1(t). 4. Solutions to Advanced Problems Let - 125 g(s)—g(t) f(s) - f(t) Let h1 = g — d1f for some d1 E R. Then h1 is monotonic. But hi(s) — h1(t) Since f(s) — = g(s) — g(t) f(t) — d1(f(s) — f(t)) = (f(s) — f(t))(u — d1). 0 is fixed, the monotonicity of h1 depends only on thesignofu—d1. Since f is nonconstant, there exist xl,x2 c R such that 1(x1) f(x2). Let c= g(xl)—g(x2) f(xi) — f(x2) = g — cf. Then r = h(xi) = h(x2) and the monotonicity of h1 = and h — d1f, for each d1, depends only on the sign of c — d1. We claim that h = g — cf is a constant function. We prove our claim by contradiction. Suppose, on the contrary, that there exists x3 c R such that h(xs) r. and f(x2) f(xi) is Since f(xi) f(x2), at least one of f(xi) true. Without loss of generality, suppose that 1(x1) Let c' = g(x1) — f(x3). g(x3) f(xi) — f(x3) Then the monotonicity of h1 also depends only on the sign of c' — d1. Since h(x3) r = h(xi), f(xi) — hence c—d1 f(x3) =c'; c' —d1. So there exists some d1 such that h1 is both strictly increasing and decreasing, which is impossible. Therefore our assumption is false and h is a constant function. Now we prove our main result. If fi f2, 13 are all constant functions, the result is trivial. Without loss of generality, suppose that Ii is nonconstant. 4. Solutions to Advanced Problems 126 For a3 = 0, we apply the lemma to Ii and 12, so 12 = cf1 +d; for a2 we apply the lemma to fi and 13, so Here c, c', d, d' are constant. 0, = c'f1 + d'. We have (c'd — cd')f1 + d'f2 — df3 = (c'd — cd')fi + d'(cfi + d) — d(c'fi + d') = If (c'd—cd',d',—d) 0. (0,0,0), then let (ci, c2, c3) = (c'd — cd', d', —d) we are done. Otherwise, d = d' = 0 and 12,13 are constant multiples of Ii Then the problem is again trivial. and Solution 99, Alternative 2 Define the vector v(x) = (1i(x),12(x),13(x)) for x C R. If the v(x) span a proper subspace of R3, we cart find a vector (ci, c2, c3) orthogonal to that subspace, and then cifi(x) + c2f2(x) + c3f3(x) = 0 for all x R. So suppose the v(x) span all of R3. Then exist x1 < X2 < X3 c R such that v(xi), v(x2), v(x3) are has linearly independent, and so the 3 x 3 matrix A with = linearly independent rows. But then A is invertible, and its columns also span R3. This means we can find c2, c3 such that = (0,1,0), and the function cif1 +c2f2 +c3f3 is then not monotonic, a contradiction. 4. Solutions to Advanced Problems 127 Problem 100 [USAMO 1999 proposal, Richard Stong] Let x1,x2,. . be variables, and let Y1,Y2,. be the sums of nonempty subsets of be the kth elementary symmetric polynomial in Let pk(xl,. the (the sum of every product of k distinct yj'S). . . . . . , For which k and n is every coefficient of Pk (as a polynomial in x1,.. even? For example, ifn = 2, then Y1,Y2,Y3 are +x2 and . , Pi =Y1+Y2+Y32X1+2X2, Y1Y2+Y2Y3+Y3Y1 P2 P3 = = Y1Y2Y3 + Solution 100 We say a polynomial pk is even if every coefficient of Pk is even. Otherwise, we say Pk is not even. For any fixed positive integer n, we say a nonnegative integer k is bad for n if k = — for some nonnegative integer j. We will show by induction on n that pk(xl, x2, is not even if and only if k is bad for ii. , . For n = = 1, x1 k=1=21—2°=r—2°. is not even and k = 1 is bad for n = Suppose that the claim is true for a certain n. We now consider pk(x1,x2, Let 0k(yl, Y2,. .. Ys) be . the . . elementary symmetric polynomial. We have the following useful, but easy to prove, facts: 1. 2. For all 1 r ,y8) 3. = i+j=k > ,Yr)Uj(Yr+1, . 0k(X+Y1,X+y2,...,X+Y&) = iI<22<"<ik = . {si Sl<82<<S,. ,ik} 1 as 4. Solutions to Advanced Problems 128 = Hence = i+j=k + +X2 + = , . , r.—O By the induction hypothesis, every term of Pr(Xi, x2 , xi,) is even un- For such r, note that 2t j - r) - - isevenunlessj—r=Oorj--r=2t. Therefore, taking coefficients modulo 2, ,Xn)Pj(X1,X2,' i+j=k ,Xn)p2n_2t(x1,X2, By the induction hypothesis, the terms in the first sum are even unless In the second sum, every term appears twice except the term Pk/2(X1, , for k even. By the induction hypothesis, this term is even unless k/2 = 2v+1 some 0 <v n, that is k = It follows that Pk(X1, x2,•• is even unless k = — 2w — for for some 4. Solutions to Advanced Problems 129 Furthermore, note that the odd coefficients in Pk(X1,X2, occur for different powers of Therefore, the condition that k is bad for n + 1 is also sufficient for to be odd. Our induction is complete. Problem 101 [Russia 20001 Prove that there exist 10 distinct real numbers a1, a2,..., a10 such that the equation (x —ai)(x—a2)...(x —a10) =(x-i--ai)(x-i-a2).(x+aio) has exactly 5 different real roots. Solution 101 We show that 6,. . —2} is a group of numbers a2,. .. a10} = satisfying the conditions given in the problem. The given equality becomes , (x — 2)(x — . , 1)x(x + 1)(x + 2)g(x2) 0, where g(u) = 2[((7+6++3)u2+ (7.6•5+7.6.4+...--5.4.3)u+76•3]. no real solutions, then g(x2) = 0 has no real solutions. If u1 and u2 are real solutions of g(u) = 0, then u1 -F-u2 <0 and U1U2 > 0, that is, both u1 and u2 are negative. It follows again that g(x2) = 0 has no real solutions. Our proof is complete. If g(u) = has GLOSSARY Arithmetic-Geometric Mean Inequality (AM—GM Inequality) If a1, a2,.. . n nonnegative are , numbers, then 1 with equality if and only if = al = a2 = Binomial Coefficient in the expansion of (x + The coefficient of is (n'\ = k!(n—k)! Cauchy-Schwarz Inequality For any with real numbers equality if a1, a2,.. . and only if and b1, b2,.. , and , are proportional, i = 1,2,. .. , n. Chebyshev Inequality 1. Let Xi, X2 . . , and yi, bers, such that x1 X2 < . . , be two sequences of real num- and Yi y2 ... yn. Then +X2 + 2. +Xn)(Yi +Y2+" +Yn) +X2y2 . Let Xl,X2...,Xn and Y1,Y2,.•.,Yn be two sequences of real numbers, such that x1 x2 .. xi-, and Yt y2 Then +yn) Xlyl+X2y2+ +XnYn. Glossary 132 De Moivre's Formula For any angle a and for any integer n, (cosa+ Elementary = cosna+isinna. Symmetric Polynomials (Functions) Given indeterminates x1, ..., Xn, the elementary symmetric functions are defined by the relation (in another indeterminate t) Si,. .. (t + x1) .. (t + + = tTh + + + Sn. taken k at a time. It That is, Sk is the sum of the products of the can be is a basic result that every symmetric polynomial in x1 (uniquely) expressed as a polynomial in the and vice versa. Fibonacci Numbers Sequence defined recursively by F1 = F2 = 1, = + for all n E N. Jensen's Inequality 1ff is concave up on an interval [a, b] and A1, A2, ..., numbers with sum equal to 1, then Aif(xi)+A2f(x2)+-.. are nonnegative +).2X2+ in the interval [a. b]. If the function is concave X2 down, the inequality is reversed. for any x1 , Lagrange's Interpolation Formula be ar. ,x7, be distinct real numbers, and let YO.Y1,. .. bitrary real numbers. Then there exists a unique polynomial P(x) of degree at most n such that i = 0. 1,.. . , n. This is the = Let Xo,Xi,. . polynomial given by — i=O x0) — — Law of Cosines Let ABC be a triangle. Then BC2 = AB2 + AC2 - 2AB . ACcosA. — Glossary 133 Lucas' Theorem Let p be a prime; let a and & be two positive integers such that + ak_lpkI + a = + a0, b = bkpk + b2 <p are integers for i = where 0 (a'\ (ak\ (ak_1\ 0, 1 + bip + b0, k. Then (aj\ (ao\ (modp). Pigeonhole Principle If n objects are distributed among k < n boxes, some box contains at least two objects. Root Mean Square-Arithmetic Mean Inequality (RMS—AM Inequality) For positive numbers x1, X2,. . . , \/X?+X2++XkXl+X2++Xk be any positive numbers for which ai + we define For positive numbers x1, x2,... More generally, let a1, a2,.. . a2 + + = 1. , , = min{x1,x2,. .. ,xk}, Moc, =max{xl,x2,...,xk}, — aj a2 0—xl x2 = + a non-zero real number. Then <M8 for s t. M00 Glossary 134 Triangle Inequality Let z = a + bi be a complex number. Define the absolute value of z to be zI= Let is and /3 be two complex numbers. The inequality called the triangle inequality. Let = Then and i9 = /9i +/32i, /92 are real numbers. where v = [/91,/92], and w = Vectors u = +192] form a triangle with sides lengths and + /31 The triangle inequality restates the fact that the length of any side of a triangle is less than the sum of the lengths of the other two sides. Vieta's Theorem Let x1, X2,. . . , be the roots of polynomial P(x) = where of the +... + + E C. Let 8k #0 and a0,a1, taken k at a time. Then . . . be + a0. the sum of the products = that is, x1x2 x1x2 + + = = a sin a cos a 1 a —; Glossary 135 addition and subtraction formulas: sin(a±b) =sinacosb±cosasinb, cos(a±b) a ± tan b tan(a±b)= tan tan a tan b' 1 double-angle formulas: sin2a = 2sinacosa, cos 2a = tan2a= cos2 a — sin2 a = 2 cos2 a — 1 = 1 — 2 tan a 1—tan2a' triple-angle formulas: sin3a = 3sina — 4s1n3 a, cos3a = 4cos3a — 3cosa, tan3 tan3a= 3tana — 2 a 1—3tan a half-angle formulas: 2 tan sina= 1 a 2 1 tan2 1 tan2 2 2 2 a+b cosa+cosb= tana +tanb = a—b sin(a+b) cosacosb' difference-to-product formulas: sin a — sin b = 2 sin a—b cos a+b 2 sin2 a, Glossary 136 cos a — cos b — —2 sin tana—tanb= a—b sin a+b sin(a — b) cos a cos b product-to-sum formulas: 2sinacosb = sin(a + b) +sin(a — b), 2cosacosb = cos(a + b) cos(a — 2sinusinb = —cos(a + b) + cos(a — b). FURTHER READING 1. Andreescu, T. Kedlaya, K.; Zeitz, P., Mathematical Contests 1995-1996: Olympiad Problems from around the World, with Solutions, American Mathematics Competitions, 1997. 2. Andreescu, T. Kedlaya, K., Mathematical Contests 1996-1997: Olympiad Problems from around the World, with Solutions, American Mathematics Competitions, 1998. 3. Andreescu, T. Kedlaya, K., Mathematical Contests 1997-1998: Olympiad Problems from around the World, with Solutions, American Mathematics Competitions, 1999. 4. Andreescu, T. Feng, Z., Mathematical Olympiads: Problems and Solutions from around the World, 1998-1999, Mathematical Association of America, 2000. 5. Andreescu, T. Gelca, R., Mathematical Olympiad Challenges, Birkhäuser, 2000. 6. Barbeau, E., Polynomials, Springer-Verlag, 1989. 7. Beckenbach, E. F., Bellman, R., An Introduction to Inequalities, New Mathematical Library, Vol. 3, Mathematical Association of America, 1961. 8. Chinn, W. C., Steenrod, N. E., First Concepts of Topology, New Mathematical Library, Vol. 27, Random House, 1966. 9. Cofman, J., What to Solve?, Oxford Science Publications, 1990. 10. Coxeter, H. S. M., Greitzer, S. L., Geometry Revisited, New Mathematical Library, Vol. 19, Mathematical Association of America, 1967. 11. Doob, M., The Canadian Mathematical Olympiad 1969-1993, University of Toronto Press, 1993. 12. Engel, A., Problem-Solving Strategies, Problem Books in Mathematics, Springer, 1998. 13. Fomin, D., Kirichenko. A., Leningrad Mathematical Olympiads 1987-1991, MathPro Press, 1994. Further Reading 138 14. Fomin, D., Genkin, S., Itenberg, 1., Mathematical Circles, American Mathematical Society, 1996. 15. Graham, R. L., Knuth, D. E., Patashnik, 0., Concrete Mathematics, Addison-Wesley, 1989. 16. Greitzer, S. L., International Mathematical Olympiads, 1959-1977, New Mathematical Library, Vol. 27, Mathematical Association of America, 1978. 17. Grossman, I., Magnus, W., Groups and Their Graphs, New Mathematical Library, Vol. 14, Mathematical Association of America, 1964. 18. Kazarinoff, N. D., Geometric Inequalities, New Mathematical Library, Vol. 4, Random House, 1961. 19. Klamkin, M., International Mathematical Olympiads, 1978-1985, New Mathematical Library, Vol. 31, Mathematical Association of America, 1986. 20. Klamkin, M., USA Mathematical Olympiads, 1972-1986, New Mathematical Library, Vol. 33, Mathematical Association of America, 1988. 21. Kiirschák, J., Hungarian Problem Book, Volumes I & II, New Mathematical Library, Vols. 11 & 12, Mathematical Association of America, 1967. 22. Kuczma, M., 144 Problems of the Austrian-Polish Mathematics Competition 1978-1998, The Academic Distribution Center, 1994. 23. Larson, L. C., Problem-Solving Through Problems, Springer-Verlag, 1983. 24. Lausch, H., Bosch Giral, C., The Asian Pacific Mathematics Olympiad 1989-2000, AMT Publishing, Canberra, 2001. 25. Liu, A., Chinese Mathematics Competitions and Olympiads 1981-1993, AMT Publishing, Canberra, 1998. 26. Lozansky, E., Rousseau, C. Winning Solutions, Springer, 1996. 27. Ore, 0., Graphs and their uses, Random House, 1963. 28. Ore, 0., Invitation to number theory, Random House, 1967. 29. Sharygin, I. F., Problems in Plane Geometry, Mir, Moscow, 1988. Further Reading 139 30. Sharygin, I. F., Problems in Solid Geometry, Mir, Moscow, 1986. 31. Shklarsky, D. 0, Chentzov, N. N; Yaglom, I. M., The USSR Olympiad Problem Book, Freeman, 1962. 32. Slinko, A., USSR Mathematical Olympiads 1989-1992, AMT Publishing, Canberra, 1997. 33. Soifer, A., Colorado Mathematical Olympiad: The first ten years, Center for excellence in mathematics education, 1994. 34. Szekely, C. J., Contests in Higher Mathematics, Springer- Verlag, 1996. 35. Stanley, R. P., Enumerative Combinatorics, Cambridge University Press, 1997. 36. Taylor, P. J., Tournament of Towns 1980-1984, AMT Publishing, Canberra, 1993. 37. Taylor, P. J., Tournament of Towns 1984-1 989, AMT Publishing, Canberra, 1992. 38. Taylor, P. J., Tournament of Towns 1989-1993, AMT Publishing, Canberra, 1994. 39. Taylor, P. .1., Storozhev, A., Tournament of Towns 1993-1997, AMT Publishing, Canberra, 1998. 40. Tomescu, I., Problems in Combinatorics and Graph Theory, Wiley 1985. 41. Vanden Eynden, C., Elementary Number Theory, McGraw-Hill, 1987. 42. Wilf, H. S., Generatingfunctionology, Academic Press, 1994. 43. Wilson, R., Introduction to graph theory, Academic Press, 1972. 44. Yaglom, 1. M., Geometric Transformations, New Mathematical Library, Vol. 8, Random House, 1962. 45. Yaglom, I. M., Geometric Transformations II, New Mathematic Library, Vol. 21, Random House, 1968. 46. Yaglom , I. M., Geometric Transformations III, New Mathemati Library, Vol. 24, Random House, 1973. 47. Zeitz, P., The Art and Craft of Problem Solving, John Wiley & Sons, 1999. Titu Andreescu is Director of the A met jean tics Competitions, serves as Head Coach of the USA international Mathem atical Olympiad (lMO) Team, is Chair of the USA Mathematical Olympiad Committee, and is Director of the USA Mathematical Olympiad Summer Program. Originally from Romania, Prof. Andreescu received the Distinguished Professor Award from the Romanian Ministty of Education in 1983, then after moving to the USA was awarded the Edyth May Sliffe Award for Distinguished High School Mathematics Teaching from the MM in 1994. in addition, he received a Certificate of Appreciation presented by the President of the MM for This outstanding service as coach of the USA Mathematical Olympiad Program in prepanng the USA team for its perfect performance in Hong Xong at the 1994 1MO". Zuming Feng graduated with a PhD from Johns Hopkins University with an on Algebraic Nuniber Theory and Elliptic Curves. He teaches at Phillips Exeter Academy. He also serves as a coach of the USA international Mathematical Olympiad (IMO) Team, is a member of the USA Mathematical Olympiad Committee, and is an assistant director of the USA Mathematical Olympiad Summer Program. He received the Edyth May Sliffe Award for THF AUSTRALIAN MATHEMATiCS TRUST Distinguished High School Mathematics Teaching from the MM in 1996. ENRICHMENT SERIES 1S8N187642012X